chapter 16 solution manual 11e

63
CHAPTER 16 ACID-BASE EQUILIBRIA AND SOLUBILITY EQUILIBRIA Problem Categories Biological: 16.16, 16.109, 16.127, 16.132, 16.137, 16.149. Conceptual: 16.19, 16.20, 16.21, 16.22, 16.37, 16.38, 16.39, 16.40, 16.71, 16.72, 16.91, 16.108, 16.123, 16.129, 16.133, 16.135, 16.142, 16.143. Descriptive: 16.38, 16.83, 16.84, 16.87, 16.89, 16.90, 16.99, 16.101, 16.117, 16.119, 16.120, 16.122, 16.124, 16.147, 16.148. Environmental: 16.130. Difficulty Level Easy: 16.5, 16.6, 16.9, 16.10, 16.11, 16.12, 16.19, 16.20, 16.39, 16.40, 16.43, 16.53, 16.55, 16.56, 16.58, 16.97, 16.101, 16.120. Medium: 16.13, 16.14, 16.15, 16.16, 16.21, 16.22, 16.23, 16.24, 16.27, 16.28, 16.29, 16.30, 16.45, 16.46, 16.54, 16.57, 16.59, 16.60, 16.61, 16.67, 16.68, 16.69, 16.10, 16.71, 16.72, 16.73, 16.74, 16.75, 16.79, 16.83, 16.84, 16.87, 16.88, 16.89, 16.90, 16.91, 16.92, 16.93, 16.95, 16.99, 16.104, 16.107, 16.108, 16.109, 16.110, 16.112, 16.117, 16.121, 16.122, 16.123, 16.124, 16.126, 16.128, 16.129, 16.131, 16.136, 16.140, 16.141, 16.142, 16.143, 16.144. Difficult: 16.17, 16.18, 16.31, 16.32, 16.33, 16.34, 16.35, 16.36, 16.37, 16.38, 16.44, 16.62, 16.63, 16.64, 16.76, 16.80, 16.81, 16.82, 16.94, 16.96, 16.98, 16.100, 16.102, 16.103, 16.105, 16.106, 16.111, 16.113, 16.114, 16.115, 16.116, 16.118, 16.119, 16.125, 16.127, 16.130, 16.132, 16.133, 16.134, 16.135, 16.137, 16.138, 16.139. 16.5 (a) This is a weak acid problem. Setting up the standard equilibrium table: CH 3 COOH(aq) H + (aq) + CH 3 COO - (aq) Initial (M): 0.40 0.00 0.00 Change (M): -x +x +x Equilibrium (M): (0.40 - x) x x 3 a 3 [H ][CH COO ] [CH COOH] + - = K 2 2 5 1.8 10 (0.40 ) 0.40 - × = - x x x x = [H + ] = 2.7 × 10 3 M pH = 2.57 (b) In addition to the acetate ion formed from the ionization of acetic acid, we also have acetate ion formed from the sodium acetate dissolving. CH 3 COONa(aq) CH 3 COO - (aq) + Na + (aq) Dissolving 0.20 M sodium acetate initially produces 0.20 M CH 3 COO - and 0.20 M Na + . The sodium ions are not involved in any further equilibrium (why?), but the acetate ions must be added to the equilibrium in part (a). CH 3 COOH(aq) H + (aq) + CH 3 COO - (aq) Initial (M): 0.40 0.00 0.20 Change (M): -x +x +x Equilibrium (M): (0.40 - x) x (0.20 + x)

Post on 22-Oct-2014

693 views

Category:

Education


17 download

DESCRIPTION

Eleventh Edition Chemistry Raymond Chang Chapter 16 Solutions

TRANSCRIPT

Page 1: Chapter 16 solution manual 11e

CHAPTER 16

ACID-BASE EQUILIBRIA AND

SOLUBILITY EQUILIBRIA Problem Categories Biological: 16.16, 16.109, 16.127, 16.132, 16.137, 16.149.

Conceptual: 16.19, 16.20, 16.21, 16.22, 16.37, 16.38, 16.39, 16.40, 16.71, 16.72, 16.91, 16.108, 16.123, 16.129,

16.133, 16.135, 16.142, 16.143.

Descriptive: 16.38, 16.83, 16.84, 16.87, 16.89, 16.90, 16.99, 16.101, 16.117, 16.119, 16.120, 16.122, 16.124, 16.147, 16.148.

Environmental: 16.130.

Difficulty Level Easy: 16.5, 16.6, 16.9, 16.10, 16.11, 16.12, 16.19, 16.20, 16.39, 16.40, 16.43, 16.53, 16.55, 16.56, 16.58, 16.97,

16.101, 16.120.

Medium: 16.13, 16.14, 16.15, 16.16, 16.21, 16.22, 16.23, 16.24, 16.27, 16.28, 16.29, 16.30, 16.45, 16.46, 16.54, 16.57,

16.59, 16.60, 16.61, 16.67, 16.68, 16.69, 16.10, 16.71, 16.72, 16.73, 16.74, 16.75, 16.79, 16.83, 16.84, 16.87, 16.88,

16.89, 16.90, 16.91, 16.92, 16.93, 16.95, 16.99, 16.104, 16.107, 16.108, 16.109, 16.110, 16.112, 16.117, 16.121,

16.122, 16.123, 16.124, 16.126, 16.128, 16.129, 16.131, 16.136, 16.140, 16.141, 16.142, 16.143, 16.144.

Difficult: 16.17, 16.18, 16.31, 16.32, 16.33, 16.34, 16.35, 16.36, 16.37, 16.38, 16.44, 16.62, 16.63, 16.64, 16.76, 16.80,

16.81, 16.82, 16.94, 16.96, 16.98, 16.100, 16.102, 16.103, 16.105, 16.106, 16.111, 16.113, 16.114, 16.115, 16.116,

16.118, 16.119, 16.125, 16.127, 16.130, 16.132, 16.133, 16.134, 16.135, 16.137, 16.138, 16.139.

16.5 (a) This is a weak acid problem. Setting up the standard equilibrium table:

CH3COOH(aq) ⇌ H+(aq) + CH3COO

−(aq)

Initial (M): 0.40 0.00 0.00

Change (M): −x +x +x

Equilibrium (M): (0.40 − x) x x

3a

3

[H ][CH COO ]

[CH COOH]

+ −

=K

2 2

51.8 10(0.40 ) 0.40

−× = ≈

x x

x

x = [H+] = 2.7 × 10

3 M

pH = 2.57

(b) In addition to the acetate ion formed from the ionization of acetic acid, we also have acetate ion formed

from the sodium acetate dissolving.

CH3COONa(aq) → CH3COO−(aq) + Na

+(aq)

Dissolving 0.20 M sodium acetate initially produces 0.20 M CH3COO− and 0.20 M Na

+. The sodium

ions are not involved in any further equilibrium (why?), but the acetate ions must be added to the

equilibrium in part (a).

CH3COOH(aq) ⇌ H+(aq) + CH3COO

−(aq)

Initial (M): 0.40 0.00 0.20

Change (M): −x +x +x

Equilibrium (M): (0.40 − x) x (0.20 + x)

Page 2: Chapter 16 solution manual 11e

CHAPTER 16: ACID-BASE EQUILIBRIA AND SOLUBILITY EQUILIBRIA 543

3a

3

[H ][CH COO ]

[CH COOH]

+ −

=K

5 ( )(0.20 ) (0.20)

1.8 10(0.40 ) 0.40

− +× = ≈

x x x

x

x = [H+] = 3.6 × 10

−5 M

pH = 4.44

Could you have predicted whether the pH should have increased or decreased after the addition of the

sodium acetate to the pure 0.40 M acetic acid in part (a)?

An alternate way to work part (b) of this problem is to use the Henderson-Hasselbalch equation.

a[conjugate base]

pH p log[acid]

= +K

5 0.20

log(1.8 10 ) log 4.74 0.300.40

−= − × + = − =pH 4.44

M

M

16.6 (a) This is a weak base calculation.

NH3(aq) + H2O(l) ⇌ 4NH ( )+aq + OH

−(aq)

Initial (M): 0.20 0 0

Change (M): −x +x +x

Equilibrium (M): 0.20 − x x x

4b

3

[NH ][OH ]

[NH ]

+ −

=K

2

5 ( )( )1.8 10

0.20 0.20

−× = ≈

x x x

x

x = 1.9 × 10−3

M = [OH−]

pOH = 2.72

pH ==== 11.28

(b) The initial concentration of 4NH+is 0.30 M from the salt NH4Cl. We set up a table as in part (a).

NH3(aq) + H2O(l) ⇌ 4NH ( )+aq + OH

−(aq)

Initial (M): 0.20 0.30 0

Change (M): −x +x +x

Equilibrium (M): 0.20 − x 0.30 + x x

4b

3

[NH ][OH ]

[NH ]

+ −

=K

5 ( )(0.30 ) (0.30)

1.8 100.20 0.20

− +× = ≈

x x x

x

x = 1.2 × 10−5

M = [OH−]

pOH = 4.92

pH ==== 9.08

Page 3: Chapter 16 solution manual 11e

CHAPTER 16: ACID-BASE EQUILIBRIA AND SOLUBILITY EQUILIBRIA 544

Alternatively, we could use the Henderson-Hasselbalch equation to solve this problem. Table 15.4

gives the value of Ka for the ammonium ion. Substituting into the Henderson-Hasselbalch equation

gives:

10

a[conjugate base] (0.20)

pH p log log(5.6 10 ) logacid (0.30)

−= + = − × +K

pH = 9.25 − 0.18 = 9.07

Is there any difference in the Henderson-Hasselbalch equation in the cases of a weak acid and its

conjugate base and a weak base and its conjugate acid?

16.9 (a) HCl (hydrochloric acid) is a strong acid. A buffer is a solution containing both a weak acid and a weak

base. Therefore, this is not a buffer system.

(b) H2SO4 (sulfuric acid) is a strong acid. A buffer is a solution containing both a weak acid and a weak

base. Therefore, this is not a buffer system.

(c) This solution contains both a weak acid, 2 4H PO−and its conjugate base,

24HPO .−

Therefore, this is a

buffer system.

(d) HNO2 (nitrous acid) is a weak acid, and its conjugate base, 2NO−(nitrite ion, the anion of the salt

KNO2), is a weak base. Therefore, this is a buffer system.

16.10 Strategy: What constitutes a buffer system? Which of the preceding solutions contains a weak acid and its

salt (containing the weak conjugate base)? Which of the preceding solutions contains a weak base and its salt

(containing the weak conjugate acid)? Why is the conjugate base of a strong acid not able to neutralize an

added acid?

Solution: The criteria for a buffer system are that we must have a weak acid and its salt (containing the

weak conjugate base) or a weak base and its salt (containing the weak conjugate acid).

(a) HCN is a weak acid, and its conjugate base, CN−, is a weak base. Therefore, this is a buffer system.

(b) 4HSO−is a weak acid, and its conjugate base,

24SO −

is a weak base (see Table 15.5 of the text).

Therefore, this is a buffer system.

(c) NH3 (ammonia) is a weak base, and its conjugate acid, 4NH+is a weak acid. Therefore, this is a buffer

system.

(d) Because HI is a strong acid, its conjugate base, I−, is an extremely weak base. This means that the I

ion will not combine with a H+ ion in solution to form HI. Thus, this system cannot act as a buffer

system.

16.11 4NH ( )+aq ⇌ NH3(aq) + H

+(aq)

Ka = 5.6 × 10−10

pKa = 9.25

3a

4

[NH ] 0.15p log 9.25 log

0.35[NH ]+= + = + =pH 8.88

MK

M

16.12 Strategy: The pH of a buffer system can be calculated in a similar manner to a weak acid equilibrium

problem. The difference is that a common-ion is present in solution. The Ka of CH3COOH is 1.8 × 10−5

(see Table 15.3 of the text).

Page 4: Chapter 16 solution manual 11e

CHAPTER 16: ACID-BASE EQUILIBRIA AND SOLUBILITY EQUILIBRIA 545

Solution: (a) We summarize the concentrations of the species at equilibrium as follows:

CH3COOH(aq) ⇌ H+(aq) + CH3COO

−(aq)

Initial (M): 2.0 0 2.0

Change (M): −x +x +x

Equilibrium (M): 2.0 − x x 2.0 + x

3a

3

[H ][CH COO ]

[CH COOH]

+ −

=K

a[H ](2.0 ) [H ](2.0)

(2.0 2.0

+ ++

= ≈−

xK

x)

Ka = [H+]

Taking the −log of both sides,

pKa = pH

Thus, for a buffer system in which the [weak acid] = [weak base],

pH = pKa

pH = −log(1.8 × 10−5

) = 4.74

(b) Similar to part (a),

pH = pKa = 4.74

Buffer (a) will be a more effective buffer because the concentrations of acid and base components are

ten times higher than those in (b). Thus, buffer (a) can neutralize 10 times more added acid or base

compared to buffer (b).

16.13 H2CO3(aq) ⇌ 3HCO ( )−aq + H

+(aq)

1

7a 4.2 10

−= ×K

1ap 6.38=K

3a

2 3

[HCO ]pH p log

[H CO ]

= +K

3

2 3

[HCO ]8.00 6.38 log

[H CO ]

= +

3

2 3

[HCO ]log 1.62

[H CO ]

=

3

2 3

[HCO ]41.7

[H CO ]

=

2 3

3

[H CO ]

[HCO ]−= 0.024

Page 5: Chapter 16 solution manual 11e

CHAPTER 16: ACID-BASE EQUILIBRIA AND SOLUBILITY EQUILIBRIA 546

16.14 Step 1: Write the equilibrium that occurs between 2 4H PO−and

24HPO .−

Set up a table relating the initial

concentrations, the change in concentration to reach equilibrium, and the equilibrium concentrations.

2 4H PO ( )−aq ⇌ H

+(aq) +

24HPO ( )−

aq

Initial (M): 0.15 0 0.10

Change (M): −x +x +x

Equilibrium (M): 0.15 − x x 0.10 + x

Step 2: Write the ionization constant expression in terms of the equilibrium concentrations. Knowing the

value of the equilibrium constant (Ka), solve for x.

24

a

2 4

[H ][HPO ]

[H PO ]

+ −

−=K

You can look up the Ka value for dihydrogen phosphate in Table 15.5 of your text.

8 ( )(0.10 )

6.2 10(0.15

− +× =

x x

x)

8 ( )(0.10)

6.2 10(0.15

−× ≈

x

)

x = [H+] = 9.3 × 10

−8 M

Step 3: Having solved for the [H+], calculate the pH of the solution.

pH = −log[H+] = −log(9.3 × 10

−8) = 7.03

16.15 Using the Henderson−Hasselbalch equation:

3a

3

[CH COO ]pH p log

[CH COOH]

= +K

3

3

[CH COO ]4.50 4.74 log

[CH COOH]

= +

Thus,

3

3

[CH COO ]

[CH COOH]

= 0.58

16.16 We can use the Henderson-Hasselbalch equation to calculate the ratio 3 2 3[HCO ] / [H CO ].− The Henderson-

Hasselbalch equation is:

a[conjugate base]

pH p log[acid]

= +K

For the buffer system of interest, 3HCO−is the conjugate base of the acid, H2CO3. We can write:

7 3

2 3

[HCO ]pH 7.40 log(4.2 10 ) log

[H CO ]

−−

= = − × +

3

2 3

[HCO ]7.40 6.38 log

[H CO ]

= +

Page 6: Chapter 16 solution manual 11e

CHAPTER 16: ACID-BASE EQUILIBRIA AND SOLUBILITY EQUILIBRIA 547

The [conjugate base]/[acid] ratio is:

3

2 3

[HCO ]log 7.40 6.38 1.02

[H CO ]

= − =

1.02

10= =13

2 3

[HCO ]1.0 10

[H CO ]

−−−−

××××

The buffer should be more effective against an added acid because ten times more base is present compared

to acid. Note that a pH of 7.40 is only a two significant figure number (Why?); the final result should only

have two significant figures.

16.17 For the first part we use Ka for ammonium ion. (Why?) The Henderson−Hasselbalch equation is

10 (0.20 )

log(5.6 10 ) log(0.20 )

−= − × + =pH 9.25

M

M

For the second part, the acid−base reaction is

NH3(g) + H+(aq) → 4NH ( )+

aq

We find the number of moles of HCl added

0.10 mol HCl

10.0 mL 0.0010 mol HCl1000 mL soln

× =

The number of moles of NH3 and 4NH+originally present are

0.20 mol

65.0 mL 0.013 mol1000 mL soln

× =

Using the acid-base reaction, we find the number of moles of NH3 and 4NH+after addition of the HCl.

NH3(aq) + H+(aq) → 4NH ( )+

aq

Initial (mol): 0.013 0.0010 0.013

Change (mol): −0.0010 −0.0010 +0.0010

Final (mol): 0.012 0 0.014

We find the new pH:

(0.012)

9.25 log(0.014)

= + =pH 9.18

16.18 As calculated in Problem 16.12, the pH of this buffer system is equal to pKa.

pH = pKa = −log(1.8 × 10−5

) = 4.74

(a) The added NaOH will react completely with the acid component of the buffer, CH3COOH. NaOH

ionizes completely; therefore, 0.080 mol of OH− are added to the buffer.

Step 1: The neutralization reaction is:

CH3COOH(aq) + OH−(aq) → CH3COO

−(aq) + H2O(l)

Initial (mol): 1.00 0.080 1.00

Change (mol): −0.080 −0.080 +0.080

Final (mol): 0.92 0 1.08

Page 7: Chapter 16 solution manual 11e

CHAPTER 16: ACID-BASE EQUILIBRIA AND SOLUBILITY EQUILIBRIA 548

Step 2: Now, the acetic acid equilibrium is reestablished. Since the volume of the solution is 1.00 L, we can

convert directly from moles to molar concentration.

CH3COOH(aq) ⇌ H+(aq) + CH3COO

−(aq)

Initial (M): 0.92 0 1.08

Change (M): −x +x +x

Equilibrium (M): 0.92 − x x 1.08 + x

Write the Ka expression, then solve for x.

3a

3

[H ][CH COO ]

[CH COOH]

+ −

=K

5 ( )(1.08 ) (1.08)

1.8 10(0.92 0.92

− +× = ≈

x x x

x)

x = [H+] = 1.5 × 10

−5 M

Step 3: Having solved for the [H+], calculate the pH of the solution.

pH = −log[H+] = −log(1.5 × 10

−5) = 4.82

The pH of the buffer increased from 4.74 to 4.82 upon addition of 0.080 mol of strong base.

(b) The added acid will react completely with the base component of the buffer, CH3COO−. HCl ionizes

completely; therefore, 0.12 mol of H+ ion are added to the buffer

Step 1: The neutralization reaction is:

CH3COO−(aq) + H

+(aq) → CH3COOH(aq)

Initial (mol): 1.00 0.12 1.00

Change (mol): −0.12 −0.12 +0.12

Final (mol): 0.88 0 1.12

Step 2: Now, the acetic acid equilibrium is reestablished. Since the volume of the solution is 1.00 L, we can

convert directly from moles to molar concentration.

CH3COOH(aq) ⇌ H+(aq) + CH3COO

−(aq)

Initial (M): 1.12 0 0.88

Change (M): −x +x +x

Equilibrium (M): 1.12 − x x 0.88 + x

Write the Ka expression, then solve for x.

3a

3

[H ][CH COO ]

[CH COOH]

+ −

=K

5 ( )(0.88 ) (0.88)

1.8 10(1.12 ) 1.12

− +× = ≈

x x x

x

x = [H+] = 2.3 × 10

−5 M

Page 8: Chapter 16 solution manual 11e

CHAPTER 16: ACID-BASE EQUILIBRIA AND SOLUBILITY EQUILIBRIA 549

Step 3: Having solved for the [H+], calculate the pH of the solution.

pH = −log[H+] = −log(2.3 × 10

−5) = 4.64

The pH of the buffer decreased from 4.74 to 4.64 upon addition of 0.12 mol of strong acid.

16.19 We write

1

3a 1.1 10

−= ×K

1ap 2.96=K

2

6a 2.5 10

−= ×K

2ap 5.60=K

In order for the buffer solution to behave effectively, the pKa of the acid component must be close to the

desired pH. Therefore, the proper buffer system is Na2A/NaHA.

16.20 Strategy: For a buffer to function effectively, the concentration of the acid component must be roughly

equal to the conjugate base component. According to Equation (16.4) of the text, when the desired pH is

close to the pKa of the acid, that is, when pH ≈ pKa,

[conjugate base]

log 0[acid]

or

[conjugate base]

1[acid]

Solution: To prepare a solution of a desired pH, we should choose a weak acid with a pKa value close to

the desired pH. Calculating the pKa for each acid:

For HA, pKa = −log(2.7 × 10−3

) = 2.57

For HB, pKa = −log(4.4 × 10−6

) = 5.36

For HC, pKa = −log(2.6 × 10−9

) = 8.59

The buffer solution with a pKa closest to the desired pH is HC. Thus, HC is the best choice to prepare a

buffer solution with pH = 8.60.

16.21 (1) (a), (b), and (c) can act as buffer systems. They contain both a weak acid and a weak base that are a

conjugate acid/base pair.

(2) (c) is the most effective buffer. It contains the greatest concentration of weak acid and weak base of the

three buffer solutions. It has a greater buffering capacity.

16.22 (1) The solutions contain a weak acid and a weak base that are a conjugate acid/base pair. These are buffer

solutions. The Henderson-Hasselbalch equation can be used to calculate the pH of each solution. The

problem states to treat each sphere as 0.1 mole. Because HA and A− are contained in the same volume,

we can plug in moles into the Henderson-Hasselbalch equation to solve for the pH of each solution.

(a) a[A ]

pH p log[HA]

= +K

0.5 mol

5.00 log0.4 mol

= + =

pH 5.10

Page 9: Chapter 16 solution manual 11e

CHAPTER 16: ACID-BASE EQUILIBRIA AND SOLUBILITY EQUILIBRIA 550

(b) 0.4 mol

5.00 log0.6 mol

= + =

pH 4.82

(c) 0.5 mol

5.00 log0.3 mol

= + =

pH 5.22

(d)

a

0.4 molpH 5.00 log

0.4 mol

pH p

= +

= = 5.00K

(2) The added acid reacts with the base component of the buffer (A−). We write out the acid-base reaction

to find the number of moles of A− and HA after addition of H

+.

A−(aq) + H

+(aq) → HA(aq)

Initial (mol): 0.5 0.1 0.4

Change (mol): −0.1 −0.1 +0.1

Final (mol): 0.4 0 0.5

Because the concentrations of the two buffer components are equal, the pH of this buffer equals its pKa

value.

We use the Henderson-Hasselbalch equation to calculate the pH of this buffer.

0.4 mol

5.00 log0.5 mol

= + =

pH 4.90

(3) The added base reacts with the acid component of the buffer (HA). We write out the acid-base reaction

to find the number of moles of HA and A− after addition of OH

−.

HA(aq) + OH−(aq) → A

−(aq) + H2O(l)

Initial (mol): 0.4 0.1 0.4

Change (mol): −0.1 −0.1 +0.1

Final (mol): 0.3 0 0.5

We use the Henderson-Hasselbalch equation to calculate the pH of this buffer.

0.5 mol

5.00 log0.3 mol

= + =

pH 5.22

16.23 The sodium hydroxide reacts with CH3COOH to produce CH3COO

−.

CH3COOH(aq) + NaOH(aq) → CH3COO−(aq) + Na

+(aq)

Initial (M): 1.8 x 1.2

Change (M): −x −x +x

Final (M): 1.8 – x 0 1.2 + x

Next, we use the Henderson-Hasselbalch equation to solve for x.

CH3COOH(aq) ⇌ H+(aq) + CH3COO

−(aq)

Ka = 1.8 × 10−5

, pKa = 4.74

Page 10: Chapter 16 solution manual 11e

CHAPTER 16: ACID-BASE EQUILIBRIA AND SOLUBILITY EQUILIBRIA 551

3

a3

[CH COO ]pH p log

[CH COOH]

= +K

1.2

5.22 4.74 log1.8

+= +

x

x

0.48 1.2

101.8

+=

x

x

x = 1.1 M

Because the solution volume is 1.0 L, the moles of NaOH that must be added are 1.1 moles.

16.24 The hydrochloric acid reacts with NH3 to produce 4NH+.

NH3(aq) + HCl(aq) → 4NH ( )+aq + Cl

−(aq)

Initial (M): 0.84 x 0.96

Change (M): −x −x +x

Final (M): 0.84 – x 0 0.96 + x

Next, we use the Henderson-Hasselbalch equation to solve for x.

4NH+(aq) ⇌ H

+(aq) + NH3(aq)

Ka = 5.6 × 10−10

, pKa = 9.25

3a

4

[NH ]pH p log

[NH ]+= +K

0.84

8.56 9.25 log0.96

−= +

+

x

x

0.69 0.84

100.96

− −=

+

x

x

x = 0.53 M

Because the solution volume is 1.0 L, the moles of HCl that must be added is 0.53 moles.

16.27 Since the acid is monoprotic, the number of moles of KOH is equal to the number of moles of acid.

0.08133 mol

Moles acid 16.4 mL 0.00133 mol1000 mL

= × =

0.2688 g

0.00133 mol= =Molar mass 202 g/mol

Page 11: Chapter 16 solution manual 11e

CHAPTER 16: ACID-BASE EQUILIBRIA AND SOLUBILITY EQUILIBRIA 552

16.28 We want to calculate the molar mass of the diprotic acid. The mass of the acid is given in the problem, so we

need to find moles of acid in order to calculate its molar mass.

The neutralization reaction is:

2KOH(aq) + H2A(aq) → K2A(aq) + 2H2O(l)

From the volume and molarity of the base needed to neutralize the acid, we can calculate the number of

moles of H2A reacted.

32

2

1 mol H A1.00 mol KOH11.1 mL KOH 5.55 10 mol H A

1000 mL 2 mol KOH

−× × = ×

We know that 0.500 g of the diprotic acid were reacted (1/10 of the 250 mL was tested). Divide the number

of grams by the number of moles to calculate the molar mass.

2

32

0.500 g H A

5.55 10 mol H A−= =

×2(H A) 90.1 g/molMMMM

16.29 The neutralization reaction is:

H2SO4(aq) + 2NaOH(aq) → Na2SO4(aq) + 2H2O(l)

Since one mole of sulfuric acid combines with two moles of sodium hydroxide, we write:

2 42 4

2 4

0.500 mol H SO 2 mol NaOHmol NaOH 12.5 mL H SO 0.0125 mol NaOH

1000 mL soln 1 mol H SO= × × =

3

0.0125 mol NaOH

50.0 10 L soln−= =

×concentration of NaOH 0.25 M

16.30 We want to calculate the molarity of the Ba(OH)2 solution. The volume of the solution is given (19.3 mL),

so we need to find the moles of Ba(OH)2 to calculate the molarity.

The neutralization reaction is:

2HCOOH + Ba(OH)2 → (HCOO)2Ba + 2H2O

22

2

mol Ba(OH)of Ba(OH)

L of Ba(OH) soln=M

given

need to find want to calculate

22

2

g H Amolar mass of H A

mol H A=

want to calculate given

need to find

Page 12: Chapter 16 solution manual 11e

CHAPTER 16: ACID-BASE EQUILIBRIA AND SOLUBILITY EQUILIBRIA 553

From the volume and molarity of HCOOH needed to neutralize Ba(OH)2, we can determine the moles of

Ba(OH)2 reacted.

32

2

1 mol Ba(OH)0.883 mol HCOOH20.4 mL HCOOH 9.01 10 mol Ba(OH)

1000 mL 2 mol HCOOH

−× × = ×

The molarity of the Ba(OH)2 solution is:

3

23

9.01 10 mol Ba(OH)

19.3 10 L

×=

×0.467 M

16.31 (a) Since the acid is monoprotic, the moles of acid equals the moles of base added.

HA(aq) + NaOH(aq) → NaA(aq) + H2O(l)

0.0633 mol

Moles acid 18.4 mL 0.00116 mol1000 mL soln

= × =

We know the mass of the unknown acid in grams and the number of moles of the unknown acid.

0.1276 g

0.00116 mol= =

2Molar mass 1.10 10 g/mol××××

(b) The number of moles of NaOH in 10.0 mL of solution is

40.0633 mol

10.0 mL 6.33 10 mol1000 mL soln

−× = ×

The neutralization reaction is:

HA(aq) + NaOH(aq) → NaA(aq) + H2O(l)

Initial (mol): 0.00116 6.33 × 10−4

0

Change (mol): −6.33 × 10−4

−6.33 × 10−4

+6.33 × 10−4

Final (mol): 5.3 × 10−4

0 6.33 × 10−4

Now, the weak acid equilibrium will be reestablished. The total volume of solution is 35.0 mL.

45.3 10 mol

[HA] 0.0150.035 L

−×

= = M

46.33 10 mol

[A ] 0.01810.035 L

−− ×

= = M

We can calculate the [H+] from the pH.

[H+] = 10

−pH = 10

−5.87 = 1.35 × 10

−6 M

HA(aq) ⇌ H+(aq) + A

−(aq)

Initial (M): 0.015 0 0.0181

Change (M): −1.35 × 10−6

+1.35 × 10−6

+1.35 × 10−6

Equilibrium (M): 0.015 1.35 × 10−6

0.0181

Page 13: Chapter 16 solution manual 11e

CHAPTER 16: ACID-BASE EQUILIBRIA AND SOLUBILITY EQUILIBRIA 554

Substitute the equilibrium concentrations into the equilibrium constant expression to solve for Ka.

6[H ][A ] (1.35 10 )(0.0181)

[HA] 0.015

+ − −×

= = =6

a 1.6 10K−−−−××××

16.32 The resulting solution is not a buffer system. There is excess NaOH and the neutralization is well past the

equivalence point.

0.167 mol

Moles NaOH 0.500 L 0.0835 mol1 L

= × =

30.100 mol

Moles CH COOH 0.500 L 0.0500 mol1 L

= × =

CH3COOH(aq) + NaOH(aq) → CH3COONa(aq) + H2O(l)

Initial (mol): 0.0500 0.0835 0

Change (mol): −0.0500 −0.0500 +0.0500

Final (mol): 0 0.0335 0.0500

The volume of the resulting solution is 1.00 L (500 mL + 500 mL = 1000 mL).

0.0335 mol

1.00 L= =[OH ] 0.0335 M

−−−−

(0.0335 0.0500) mol

1.00 L

+= =[Na ] 0.0835++++ M

14

w 1.0 10

0.0335[OH ]

×= = =

13[H ] 3.0 10

KM

+ −+ −+ −+ −××××

0.0500 mol

1.00 L= =3[CH COO ] 0.0500 M

−−−−

CH3COO−(aq) + H2O(l) ⇌ CH3COOH(aq) + OH

−(aq)

Initial (M): 0.0500 0 0.0335

Change (M): −x +x +x

Equilibrium (M): 0.0500 − x x 0.0335 + x

3b

3

[CH COOH][OH ]

[CH COO ]

−=K

10 ( )(0.0335 ) ( )(0.0335)

5.6 10(0.0500 ) (0.0500)

− +× = ≈

x x x

x

x = [CH3COOH] = 8.4 ×××× 10−−−−10

M

16.33 HCl(aq) + CH3NH2(aq) ⇌ 3 3CH NH ( )+aq + Cl

−(aq)

Since the concentrations of acid and base are equal, equal volumes of each solution will need to be added to

reach the equivalence point. Therefore, the solution volume is doubled at the equivalence point, and the

concentration of the conjugate acid from the salt, 3 3CH NH ,+is:

0.20

0.102

=M

M

Page 14: Chapter 16 solution manual 11e

CHAPTER 16: ACID-BASE EQUILIBRIA AND SOLUBILITY EQUILIBRIA 555

The conjugate acid undergoes hydrolysis.

3 3CH NH ( )+aq + H2O(l) ⇌ H3O

+(aq) + CH3NH2(aq)

Initial (M): 0.10 0 0

Change (M): −x +x +x

Equilibrium (M): 0.10 − x x x

3 3 2a

3 3

[H O ][CH NH ]

[CH NH ]

+

+=K

2

112.3 100.10

−× =

x

x

Assuming that, 0.10 − x ≈ 0.10

x = [H3O+] = 1.5 × 10

−6 M

pH = 5.82

16.34 Let's assume we react 1 L of HCOOH with 1 L of NaOH.

HCOOH(aq) + NaOH(aq) → HCOONa(aq) + H2O(l)

Initial (mol): 0.10 0.10 0

Change (mol): −0.10 −0.10 +0.10

Final (mol): 0 0 0.10

The solution volume has doubled (1 L + 1 L = 2 L). The concentration of HCOONa is:

0.10 mol

(HCOONa) 0.0502 L

= =M M

HCOO−(aq) is a weak base. The hydrolysis is:

HCOO−(aq) + H2O(l) ⇌ HCOOH(aq) + OH

−(aq)

Initial (M): 0.050 0 0

Change (M): −x +x +x

Equilibrium (M): 0.050 − x x x

b[HCOOH][OH ]

[HCOO ]

−=K

2 2

115.9 100.050 0.050

−× = ≈

x x

x

x = 1.7 × 10−6

M = [OH−]

pOH = 5.77

pH ==== 8.23

Page 15: Chapter 16 solution manual 11e

CHAPTER 16: ACID-BASE EQUILIBRIA AND SOLUBILITY EQUILIBRIA 556

16.35 The reaction between CH3COOH and KOH is:

CH3COOH(aq) + KOH(aq) → CH3COOK(aq) + H2O(l)

We see that 1 mole CH3COOH ≏ 1 mol KOH. Therefore, at every stage of titration, we can calculate the

number of moles of acid reacting with base, and the pH of the solution is determined by the excess acid or

base left over. At the equivalence point, however, the neutralization is complete, and the pH of the solution

will depend on the extent of the hydrolysis of the salt formed, which is CH3COOK.

(a) No KOH has been added. This is a weak acid calculation.

CH3COOH(aq) + H2O(l) ⇌ H3O+(aq) + CH3COO

−(aq)

Initial (M): 0.100 0 0

Change (M): −x +x +x

Equilibrium (M): 0.100 − x x x

3 3a

3

[H O ][CH COO ]

[CH COOH]

+ −

=K

5 ( )( )1.8 10

0.100 0.100

−× = ≈

2x x x

x

x = 1.34 × 10−3

M = [H3O+]

pH ==== 2.87

(b) The number of moles of CH3COOH originally present in 25.0 mL of solution is:

33

3

0.100 mol CH COOH25.0 mL 2.50 10 mol

1000 mL CH COOH soln

−× = ×

The number of moles of KOH in 5.0 mL is:

30.200 mol KOH

5.0 mL 1.00 10 mol1000 mL KOH soln

−× = ×

We work with moles at this point because when two solutions are mixed, the solution volume increases.

As the solution volume increases, molarity will change, but the number of moles will remain the same.

The changes in number of moles are summarized.

CH3COOH(aq) + KOH(aq) → CH3COOK(aq) + H2O(l)

Initial (mol): 2.50 × 10−3

1.00 × 10−3

0

Change (mol): −1.00 × 10−3

−1.00 × 10−3

+1.00 × 10−3

Final (mol): 1.50 × 10−3

0 1.00 × 10−3

At this stage, we have a buffer system made up of CH3COOH and CH3COO− (from the salt,

CH3COOK). We use the Henderson-Hasselbalch equation to calculate the pH.

a[conjugate base]

pH p log[acid]

= +K

3

5

3

1.00 10pH log(1.8 10 ) log

1.50 10

−−

×= − × +

×

pH = 4.56

Page 16: Chapter 16 solution manual 11e

CHAPTER 16: ACID-BASE EQUILIBRIA AND SOLUBILITY EQUILIBRIA 557

(c) This part is solved similarly to part (b).

The number of moles of KOH in 10.0 mL is:

30.200 mol KOH

10.0 mL 2.00 10 mol1000 mL KOH soln

−× = ×

The changes in number of moles are summarized.

CH3COOH(aq) + KOH(aq) → CH3COOK(aq) + H2O(l)

Initial (mol): 2.50 × 10−3

2.00 × 10−3

0

Change (mol): −2.00 × 10−3

−2.00 × 10−3

+2.00 × 10−3

Final (mol): 0.50 × 10−3

0 2.00 × 10−3

At this stage, we have a buffer system made up of CH3COOH and CH3COO− (from the salt,

CH3COOK). We use the Henderson-Hasselbalch equation to calculate the pH.

a[conjugate base]

pH p log[acid]

= +K

3

5

3

2.00 10pH log(1.8 10 ) log

0.50 10

−−

×= − × +

×

pH = 5.34

(d) We have reached the equivalence point of the titration. 2.50 × 10−3

mole of CH3COOH reacts with

2.50 × 10−3

mole KOH to produce 2.50 × 10−3

mole of CH3COOK. The only major species present in

solution at the equivalence point is the salt, CH3COOK, which contains the conjugate base, CH3COO−.

Let's calculate the molarity of CH3COO−. The volume of the solution is: (25.0 mL + 12.5 mL =

37.5 mL = 0.0375 L).

3

32.50 10 mol

(CH COO ) 0.06670.0375 L

−− ×

= =M M

We set up the hydrolysis of CH3COO−, which is a weak base.

CH3COO−(aq) + H2O(l) ⇌ CH3COOH(aq) + OH

−(aq)

Initial (M): 0.0667 0 0

Change (M): −x +x +x

Equilibrium (M): 0.0667 − x x x

3b

3

[CH COOH][OH ]

[CH COO ]

−=K

10 ( )( )5.6 10

0.0667 0.0667

−× = ≈

2x x x

x

x = 6.1 × 10−6

M = [OH−]

pOH = 5.21

pH ==== 8.79

Page 17: Chapter 16 solution manual 11e

CHAPTER 16: ACID-BASE EQUILIBRIA AND SOLUBILITY EQUILIBRIA 558

(e) We have passed the equivalence point of the titration. The excess strong base, KOH, will determine the

pH at this point. The moles of KOH in 15.0 mL are:

30.200 mol KOH

15.0 mL 3.00 10 mol1000 mL KOH soln

−× = ×

The changes in number of moles are summarized.

CH3COOH(aq) + KOH(aq) → CH3COOK(aq) + H2O(l)

Initial (mol): 2.50 × 10−3

3.00 × 10−3

0

Change (mol): −2.50 × 10−3

−2.50 × 10−3

+2.50 × 10−3

Final (mol): 0 0.50 × 10−3

2.50 × 10−3

Let's calculate the molarity of the KOH in solution. The volume of the solution is now 40.0 mL = 0.0400 L.

30.50 10 mol

(KOH) 0.01250.0400 L

−×

= =M M

KOH is a strong base. The pOH is:

pOH = −log(0.0125) = 1.90

pH = 12.10

16.36 The reaction between NH3 and HCl is:

NH3(aq) + HCl(aq) → NH4Cl(aq)

We see that 1 mole NH3 ≏ 1 mol HCl. Therefore, at every stage of titration, we can calculate the number of

moles of base reacting with acid, and the pH of the solution is determined by the excess base or acid left over.

At the equivalence point, however, the neutralization is complete, and the pH of the solution will depend on

the extent of the hydrolysis of the salt formed, which is NH4Cl.

(a) No HCl has been added. This is a weak base calculation.

NH3(aq) + H2O(l) ⇌ 4NH ( )+aq + OH

−(aq)

Initial (M): 0.300 0 0

Change (M): −x +x +x

Equilibrium (M): 0.300 − x x x

4b

3

[NH ][OH ]

[NH ]

+ −

=K

5 ( )( )1.8 10

0.300 0.300

−× = ≈

2x x x

x

x = 2.3 × 10−3

M = [OH−]

pOH = 2.64

pH ==== 11.36

(b) The number of moles of NH3 originally present in 10.0 mL of solution is:

33

3

0.300 mol NH10.0 mL 3.00 10 mol

1000 mL NH soln

−× = ×

Page 18: Chapter 16 solution manual 11e

CHAPTER 16: ACID-BASE EQUILIBRIA AND SOLUBILITY EQUILIBRIA 559

The number of moles of HCl in 10.0 mL is:

30.100 mol HCl

10.0 mL 1.00 10 mol1000 mL HClsoln

−× = ×

We work with moles at this point because when two solutions are mixed, the solution volume increases.

As the solution volume increases, molarity will change, but the number of moles will remain the same.

The changes in number of moles are summarized.

NH3(aq) + HCl(aq) → NH4Cl(aq)

Initial (mol): 3.00 × 10−3

1.00 × 10−3

0

Change (mol): −1.00 × 10−3

−1.00 × 10−3

+1.00 × 10−3

Final (mol): 2.00 × 10−3

0 1.00 × 10−3

At this stage, we have a buffer system made up of NH3 and 4NH+(from the salt, NH4Cl). We use the

Henderson-Hasselbalch equation to calculate the pH.

a[conjugate base]

pH p log[acid]

= +K

3

10

3

2.00 10pH log(5.6 10 ) log

1.00 10

−−

×= − × +

×

pH = 9.55

(c) This part is solved similarly to part (b).

The number of moles of HCl in 20.0 mL is:

30.100 mol HCl

20.0 mL 2.00 10 mol1000 mL HClsoln

−× = ×

The changes in number of moles are summarized.

NH3(aq) + HCl(aq) → NH4Cl(aq)

Initial (mol): 3.00 × 10−3

2.00 × 10−3

0

Change (mol): −2.00 × 10−3

−2.00 × 10−3

+2.00 × 10−3

Final (mol): 1.00 × 10−3

0 2.00 × 10−3

At this stage, we have a buffer system made up of NH3 and 4NH+(from the salt, NH4Cl). We use the

Henderson-Hasselbalch equation to calculate the pH.

a[conjugate base]

pH p log[acid]

= +K

3

10

3

1.00 10pH log(5.6 10 ) log

2.00 10

−−

×= − × +

×

pH = 8.95

(d) We have reached the equivalence point of the titration. 3.00 × 10−3

mole of NH3 reacts with 3.00 × 10−3

mole HCl to produce 3.00 × 10−3

mole of NH4Cl. The only major species present in solution at the

equivalence point is the salt, NH4Cl, which contains the conjugate acid, 4NH .+ Let's calculate the

molarity of 4NH .+ The volume of the solution is: (10.0 mL + 30.0 mL = 40.0 mL = 0.0400 L).

Page 19: Chapter 16 solution manual 11e

CHAPTER 16: ACID-BASE EQUILIBRIA AND SOLUBILITY EQUILIBRIA 560

3

43.00 10 mol

(NH ) 0.07500.0400 L

−+ ×

= =M M

We set up the hydrolysis of NH4+, which is a weak acid.

4NH ( )+aq + H2O(l) ⇌ H3O

+(aq) + NH3(aq)

Initial (M): 0.0750 0 0

Change (M): −x +x +x

Equilibrium (M): 0.0750 − x x x

3 3a

4

[H O ][NH ]

[NH ]

+

+=K

10 ( )( )5.6 10

0.0750 0.0750

−× = ≈

2x x x

x

x = 6.5 × 10−6

M = [H3O+]

pH ==== 5.19

(e) We have passed the equivalence point of the titration. The excess strong acid, HCl, will determine the

pH at this point. The moles of HCl in 40.0 mL are:

30.100 mol HCl

40.0 mL 4.00 10 mol1000 mL HClsoln

−× = ×

The changes in number of moles are summarized.

NH3(aq) + HCl(aq) → NH4Cl(aq)

Initial (mol): 3.00 × 10−3

4.00 × 10−3

0

Change (mol): −3.00 × 10−3

−3.00 × 10−3

+3.00 × 10−3

Final (mol): 0 1.00 × 10−3

3.00 × 10−3

Let's calculate the molarity of the HCl in solution. The volume of the solution is now 50.0 mL = 0.0500 L.

31.00 10 mol

(HCl) 0.02000.0500 L

−×

= =M M

HCl is a strong acid. The pH is:

pH = −log(0.0200) = 1.70

16.37 (1) Before any NaOH is added, there would only be acid molecules in solution − diagram (c).

(2) At the halfway-point, there would be equal amounts of acid and its conjugate base − diagram (d).

(3) At the equivalence point, there is only salt dissolved in water. In the diagram, Na+ and H2O are not

shown, so the only species present would be A− − diagram (b).

(4) Beyond the equivalence point, excess hydroxide would be present in solution − diagram (a).

The pH is greater than 7 at the equivalence point of a titration of a weak acid with a strong base like NaOH.

Page 20: Chapter 16 solution manual 11e

CHAPTER 16: ACID-BASE EQUILIBRIA AND SOLUBILITY EQUILIBRIA 561

16.38 (1) Before any HCl is added, there would only be base molecules in solution − diagram (c).

(2) At the halfway-point, there would be equal amounts of base and its conjugate acid − diagram (a).

(3) At the equivalence point, there is only salt dissolved in water. In the diagram, Cl− and H2O are not

shown, so the only species present would be BH+ − diagram (d).

(4) Beyond the equivalence point, excess hydronium ions would be present in solution − diagram (b).

The pH is less than 7 at the equivalence point of a titration of a weak base with a strong acid like HCl.

16.39 The pH at the half-way point of a weak acid/strong base titration equals the pKa value of the weak acid.

Hence, the [H+] equals the Ka value of the acid.

[H+] = Ka = 4.5 × 10

−4 M

16.40 The pH at the half-way point of a weak acid/strong base titration equals the pKa value of the weak acid. The

12.35 mL point of the titration is the half-way point. Hence, pH = pKa = 5.22. The Ka value of acid is:

ap 5.2210 10− −

= = =6

a 6.0 10K

K−−−−××××

16.43 (a) HCOOH is a weak acid and NaOH is a strong base. Suitable indicators are cresol red and

phenolphthalein.

(b) HCl is a strong acid and KOH is a strong base. Suitable indicators are all those listed with the

exceptions of thymol blue, bromophenol blue, and methyl orange.

(c) HNO3 is a strong acid and CH3NH2 is a weak base. Suitable indicators are bromophenol blue, methyl

orange, methyl red, and chlorophenol blue.

16.44 CO2 in the air dissolves in the solution:

CO2 + H2O ⇌ H2CO3

The carbonic acid neutralizes the NaOH.

16.45 The weak acid equilibrium is

HIn(aq) ⇌ H+(aq) + In

−(aq)

We can write a Ka expression for this equilibrium.

a[H ][In ]

[HIn]

+ −

=K

Rearranging,

a

[HIn] [H ]=

[In ]

+

− K

From the pH, we can calculate the H+ concentration.

[H+] = 10

−pH = 10

−4.00 = 1.0 × 10

−4 M

Page 21: Chapter 16 solution manual 11e

CHAPTER 16: ACID-BASE EQUILIBRIA AND SOLUBILITY EQUILIBRIA 562

4

6a

[H ] 1.0 10

1.0 10

+ −

×= = =

×

[HIn]100

[In ] K−−−−

Since the concentration of HIn is 100 times greater than the concentration of In−, the color of the solution will

be that of HIn, the nonionized formed. The color of the solution will be red.

16.46 According to Section 16.5 of the text, when [HIn] ≈ [In

−] the indicator color is a mixture of the colors of HIn

and In−. In other words, the indicator color changes at this point. When [HIn] ≈ [In

−] we can write:

a[In ]1

[HIn] [H ]

+= =

K

[H+] = Ka = 2.0 × 10

−6

pH ==== 5.70

16.53 (a) The solubility equilibrium is given by the equation

AgI(s) ⇌ Ag+(aq) + I

−(aq)

The expression for Ksp is given by

Ksp = [Ag+][I

−]

The value of Ksp can be found in Table 16.2 of the text. If the equilibrium concentration of silver ion is

the value given, the concentration of iodide ion must be

17

sp

9

8.3 10

[Ag ] 9.1 10

−− −

+ −

×= = = ×

×

9[I ] 9.1 10

KM

(b) The value of Ksp for aluminum hydroxide can be found in Table 16.2 of the text. The equilibrium

expressions are:

Al(OH)3(s) ⇌ Al3+

(aq) + 3OH−(aq)

Ksp = [Al3+

][OH−]3

Using the given value of the hydroxide ion concentration, the equilibrium concentration of aluminum

ion is:

33

sp

3 9 3

1.8 10

[OH ] (2.9 10 )

−+ −

− −

×= = = ×

×

3 8[Al ] 7.4 10

KM

What is the pH of this solution? Will the aluminum concentration change if the pH is altered?

16.54 Strategy: In each part, we can calculate the number of moles of compound dissolved in one liter of solution

(the molar solubility). Then, from the molar solubility, s, we can determine Ksp.

Solution:

(a)

242 2

2

7.3 10 g SrF 1 mol SrF5.8 10 mol/L =

1 L soln 125.6 g SrF

−−×

× = × s

Page 22: Chapter 16 solution manual 11e

CHAPTER 16: ACID-BASE EQUILIBRIA AND SOLUBILITY EQUILIBRIA 563

Consider the dissociation of SrF2 in water. Let s be the molar solubility of SrF2.

SrF2(s) ⇌ Sr2+

(aq) + 2F−(aq)

Initial (M): 0 0

Change (M): −s +s +2s

Equilibrium (M): s 2s

Ksp = [Sr2+

][F−]2 = (s)(2s)

2 = 4s

3

The molar solubility (s) was calculated above. Substitute into the equilibrium constant expression to

solve for Ksp.

Ksp = [Sr2+

][F−]2 = 4s

3 = 4(5.8 × 10

−4)3 = 7.8 ×××× 10

−−−−10

(b)

353 4 3 4

3 4

6.7 10 g Ag PO 1 mol Ag PO1.6 10 mol/L =

1 L soln 418.7 g Ag PO

−−×

× = × s

(b) is solved in a similar manner to (a)

The equilibrium equation is:

Ag3PO4(s) ⇌ 3Ag+(aq) +

34PO ( )−

aq

Initial (M): 0 0

Change (M): −s +3s +s

Equilibrium (M): 3s s

Ksp = [Ag+]3 3

4[PO ]− = (3s)

3(s) = 27s

4 = 27(1.6 × 10

−5)4 = 1.8 ×××× 10

−−−−18

16.55 For MnCO3 dissolving, we write

MnCO3(s) ⇌ Mn2+

(aq) +23CO ( )−

aq

For every mole of MnCO3 that dissolves, one mole of Mn2+

will be produced and one mole of23CO −

will be

produced. If the molar solubility of MnCO3 is s mol/L, then the concentrations of Mn2+

and23CO −

are:

[Mn2+

] = 23[CO ]−

= s = 4.2 × 10−6

M

Ksp = [Mn2+

]23[CO ]−

= s2 = (4.2 × 10

−6)2 = 1.8 ×××× 10

−−−−11

16.56 First, we can convert the solubility of MX in g/L to mol/L.

3

54.63 10 g MX 1 mol MX1.34 10 mol/L (molar solubility)

1 L soln 346 g MX

−−×

× = × = s

The equilibrium reaction is:

MX(s) ⇌ Mn+

(aq) + Xn−

(aq)

Initial (M): 0 0

Change (M): −s +s +s

Equilibrium (M): s s

Ksp = [Mn+

][Xn−

] = s2 = (1.34 × 10

−5)2 = 1.80 ×××× 10

−−−−10

Page 23: Chapter 16 solution manual 11e

CHAPTER 16: ACID-BASE EQUILIBRIA AND SOLUBILITY EQUILIBRIA 564

16.57 The charges of the M and X ions are +3 and −2, respectively (are other values possible?). We first calculate

the number of moles of M2X3 that dissolve in 1.0 L of water. We carry an additional significant figure

throughout this calculation to minimize rounding errors.

17 19

2 31 mol

Moles M X (3.6 10 g) 1.25 10 mol288 g

− −= × × = ×

The molar solubility, s, of the compound is therefore 1.3 × 10−19

M. At equilibrium the concentration of M3+

must be 2s and that of X2−

must be 3s. (See Table 16.3 of the text.)

Ksp = [M3+

]2[X

2−]3 = [2s]

2[3s]

3 = 108s

5

Since these are equilibrium concentrations, the value of Ksp can be found by simple substitution

Ksp = 108s5 = 108(1.25 × 10

−19)5 = 3.3 ×××× 10

−−−−93

16.58 Strategy: We can look up the Ksp value of CaF2 in Table 16.2 of the text. Then, setting up the dissociation

equilibrium of CaF2 in water, we can solve for the molar solubility, s.

Solution: Consider the dissociation of CaF2 in water.

CaF2(s) ⇌ Ca2+

(aq) + 2F−(aq)

Initial (M): 0 0

Change (M): −s +s +2s

Equilibrium (M): s 2s

Recall, that the concentration of a pure solid does not enter into an equilibrium constant expression.

Therefore, the concentration of CaF2 is not important.

Substitute the value of Ksp and the concentrations of Ca2+

and F− in terms of s into the solubility product

expression to solve for s, the molar solubility.

Ksp = [Ca2+

][F−]2

4.0 × 10−11

= (s)(2s)2

4.0 × 10−11

= 4s3

s = molar solubility = 2.2 ×××× 10−−−−4

mol/L

The molar solubility indicates that 2.2 × 10−4

mol of CaF2 will dissolve in 1 L of an aqueous solution.

16.59 Let s be the molar solubility of Zn(OH)2. The equilibrium concentrations of the ions are then

[Zn2+

] = s and [OH−] = 2s

Ksp = [Zn2+

][OH−]2 = (s)(2s)

2 = 4s

3 = 1.8 × 10

−14

1

14 351.8 10

1.7 104

−−

×= = ×

s

[OH−] = 2s = 3.4 × 10

−5 M and pOH = 4.47

pH = 14.00 − 4.47 = 9.53

Page 24: Chapter 16 solution manual 11e

CHAPTER 16: ACID-BASE EQUILIBRIA AND SOLUBILITY EQUILIBRIA 565

If the Ksp of Zn(OH)2 were smaller by many more powers of ten, would 2s still be the hydroxide ion

concentration in the solution?

16.60 First we can calculate the OH

− concentration from the pH.

pOH = 14.00 − pH

pOH = 14.00 − 9.68 = 4.32

[OH−] = 10

−pOH = 10

−4.32 = 4.8 × 10

−5 M

The equilibrium equation is:

MOH(s) ⇌ M+(aq) + OH

−(aq)

From the balanced equation we know that [M+] = [OH

−]

Ksp = [M+][OH

−] = (4.8 × 10

−5)2 = 2.3 ×××× 10

−−−−9

16.61 According to the solubility rules, the only precipitate that might form is BaCO3.

Ba2+

(aq) +23CO ( )−

aq → BaCO3(s)

The number of moles of Ba2+

present in the original 20.0 mL of Ba(NO3)2 solution is

2

3 20.10 mol Ba20.0 mL 2.0 10 mol Ba

1000 mL soln

+− +

× = ×

The total volume after combining the two solutions is 70.0 mL. The concentration of Ba2+

in 70 mL is

3 2

2 2

3

2.0 10 mol Ba[Ba ] 2.9 10

70.0 10 L

− ++ −

×= = ×

×M

The number of moles of23CO −

present in the original 50.0 mL Na2CO3 solution is

23 23

3

0.10 mol CO50.0 mL 5.0 10 mol CO

1000 mL soln

−− −

× = ×

The concentration of23CO −

in the 70.0 mL of combined solution is

3 2

2 233 3

5.0 10 mol CO[CO ] 7.1 10

70.0 10 L

− −− −

×= = ×

×M

Now we must compare Q and Ksp. From Table 16.2 of the text, the Ksp for BaCO3 is 8.1 × 10−9

. As for Q,

Q = [Ba2+

]023 0[CO ]−

= (2.9 × 10−2

)(7.1 × 10−2

) = 2.1 × 10−3

Since (2.1 × 10−3

) > (8.1 × 10−9

), then Q > Ksp. Therefore, BaCO3 will precipitate.

Page 25: Chapter 16 solution manual 11e

CHAPTER 16: ACID-BASE EQUILIBRIA AND SOLUBILITY EQUILIBRIA 566

16.62 The net ionic equation is:

Sr2+

(aq) + 2F−(aq) → SrF2(s)

Let’s find the limiting reagent in the precipitation reaction.

0.060 mol

Moles F 75 mL 0.0045 mol1000 mL soln

−= × =

2 0.15 mol

Moles Sr 25 mL 0.0038 mol1000 mL soln

+= × =

From the stoichiometry of the balanced equation, twice as many moles of F− are required to react with Sr

2+.

This would require 0.0076 mol of F−, but we only have 0.0045 mol. Thus, F

− is the limiting reagent.

Let’s assume that the above reaction goes to completion. Then, we will consider the equilibrium that is

established when SrF2 partially dissociates into ions.

Sr2+

(aq) + 2F−(aq) → SrF2(s)

Initial (mol): 0.0038 0.0045 0

Change (mol): −0.00225 −0.0045 +0.00225

Final (mol): 0.00155 0 0.00225

Now, let’s establish the equilibrium reaction. The total volume of the solution is 100 mL = 0.100 L. Divide

the above moles by 0.100 L to convert to molar concentration.

SrF2(s) ⇌ Sr2+

(aq) + 2F−(aq)

Initial (M): 0.0225 0.0155 0

Change (M): −s +s +2s

Equilibrium (M): 0.0225 − s 0.0155 + s 2s

Write the solubility product expression, then solve for s.

Ksp = [Sr2+

][F−]2

2.0 × 10−10

= (0.0155 + s)(2s)2 ≈ (0.0155)(2s)

2

s = 5.7 × 10−5

M

[F−−−−] = 2s = 1.1 ×××× 10

−−−−4 M

[Sr2++++

] = 0.0155 + s = 0.016 M

Both sodium ions and nitrate ions are spectator ions and therefore do not enter into the precipitation reaction.

2(0.0038) mol

0.10 L= =3[NO ] 0.076 M

−−−−

0.0045 mol

0.10 L= =[Na ] 0.045 M

++++

Page 26: Chapter 16 solution manual 11e

CHAPTER 16: ACID-BASE EQUILIBRIA AND SOLUBILITY EQUILIBRIA 567

16.63 (a) The solubility product expressions for both substances have exactly the same mathematical form and are

therefore directly comparable. The substance having the smaller Ksp (AgI) will precipitate first. (Why?)

(b) When CuI just begins to precipitate the solubility product expression will just equal Ksp (saturated

solution). The concentration of Cu+ at this point is 0.010 M (given in the problem), so the concentration

of iodide ion must be:

Ksp = [Cu+][I

−] = (0.010)[I

−] = 5.1 × 10

−12

12

105.1 10[I ] 5.1 10

0.010

−− −×

= = × M

Using this value of [I−], we find the silver ion concentration

17

sp

10

8.3 10

[I ] 5.1 10

− −

×= = =

×

7[Ag ] 1.6 10

K+ −+ −+ −+ −

×××× M

(c) The percent of silver ion remaining in solution is:

71.6 10

100% or0.010

−×

= × =3% Ag ( ) 0.0016% 1.6 10 %

M

M

+ −+ −+ −+ −××××aq

Is this an effective way to separate silver from copper?

16.64 For Fe(OH)3, Ksp = 1.1 × 10

−36. When [Fe

3+] = 0.010 M, the [OH

−] value is:

Ksp = [Fe3+

][OH−]3

or

1

3sp

3[OH ]

[Fe ]

+

=

K

1

36 3121.1 10

[OH ] 4.8 100.010

−− −

×= = ×

M

This [OH−] corresponds to a pH of 2.68. In other words, Fe(OH)3 will begin to precipitate from this solution

at pH of 2.68.

For Zn(OH)2, Ksp = 1.8 × 10−14

. When [Zn2+

] = 0.010 M, the [OH−] value is:

1

2sp

2[OH ]

[Zn ]

+

=

K

1

14 261.8 10

[OH ] 1.3 100.010

−− −

×= = ×

M

This corresponds to a pH of 8.11. In other words Zn(OH)2 will begin to precipitate from the solution at

pH = 8.11.

Page 27: Chapter 16 solution manual 11e

CHAPTER 16: ACID-BASE EQUILIBRIA AND SOLUBILITY EQUILIBRIA 568

We assume a 99% precipitation of Fe(OH)3 before Zn(OH)2 begins to precipitate. The Fe3+

ions remaining in

solution have a concentration given by:

(1.00 − 0.99) × 0.010 M = 1.0 × 10−4

M

Ksp = [Fe3+

][OH−]3

1.1 × 10−36

= (1.0 × 10−4

)[OH−]3

[OH−] = 2.2 × 10

−11 M

pOH = 10.66 pH = 3.34

So, the pH range over which Fe(OH)3 is practically all removed and Zn(OH)2 has not begun to precipitate is:

3.34 < pH < 8.11

16.67 First let s be the molar solubility of CaCO3 in this solution.

CaCO3(s) ⇌ Ca2+

(aq) + 23CO ( )−

aq

Initial (M): 0.050 0

Change (M): −s +s +s

Equilibrium (M): (0.050 + s) s

Ksp = [Ca2+

]23[CO ]−

= (0.050 + s)s = 8.7 × 10−9

We can assume 0.050 + s ≈ 0.050, then

9

78.7 101.7 10

0.050

−−×

= = ×s M

The mass of CaCO3 can then be found.

7

2 3100.1 g CaCO1.7 10 mol(3.0 10 mL)

1000 mL soln 1 mol

−×

× × × =6

35.1 10 g CaCO−−−−××××

16.68 Strategy: In parts (b) and (c), this is a common-ion problem. In part (b), the common ion is Br

−, which is

supplied by both PbBr2 and KBr. Remember that the presence of a common ion will affect only the solubility

of PbBr2, but not the Ksp value because it is an equilibrium constant. In part (c), the common ion is Pb2+

,

which is supplied by both PbBr2 and Pb(NO3)2.

Solution: (a) Set up a table to find the equilibrium concentrations in pure water.

PbBr2(s) ⇌ Pb2+

(aq) + 2Br−(aq)

Initial (M) 0 0

Change (M) −s +s +2s

Equilibrium (M) s 2s

Ksp = [Pb2+

][Br−]2

8.9 × 10−6

= (s)(2s)2

s = molar solubility = 0.013 M

Page 28: Chapter 16 solution manual 11e

CHAPTER 16: ACID-BASE EQUILIBRIA AND SOLUBILITY EQUILIBRIA 569

(b) Set up a table to find the equilibrium concentrations in 0.20 M KBr. KBr is a soluble salt that ionizes

completely giving an initial concentration of Br− = 0.20 M.

PbBr2(s) ⇌ Pb2+

(aq) + 2Br−(aq)

Initial (M) 0 0.20

Change (M) −s +s +2s

Equilibrium (M) s 0.20 + 2s

Ksp = [Pb2+

][Br−]2

8.9 × 10−6

= (s)(0.20 + 2s)2

8.9 × 10−6

≈ (s)(0.20)2

s = molar solubility = 2.2 ×××× 10−−−−4

M

Thus, the molar solubility of PbBr2 is reduced from 0.013 M to 2.2 × 10−4

M as a result of the common

ion (Br−) effect.

(c) Set up a table to find the equilibrium concentrations in 0.20 M Pb(NO3)2. Pb(NO3)2 is a soluble salt

that dissociates completely giving an initial concentration of [Pb2+

] = 0.20 M.

PbBr2(s) ⇌ Pb2+

(aq) + 2Br−(aq)

Initial (M): 0.20 0

Change (M): −s +s +2s

Equilibrium (M): 0.20 + s 2s

Ksp = [Pb2+

][Br−]2

8.9 × 10−6

= (0.20 + s)(2s)2

8.9 × 10−6

≈ (0.20)(2s)2

s = molar solubility = 3.3 ×××× 10−−−−3

M

Thus, the molar solubility of PbBr2 is reduced from 0.013 M to 3.3 × 10−3

M as a result of the common

ion (Pb2+

) effect.

Check: You should also be able to predict the decrease in solubility due to a common-ion using

Le Châtelier's principle. Adding Br− or Pb

2+ ions shifts the system to the left, thus decreasing the solubility

of PbBr2.

16.69 We first calculate the concentration of chloride ion in the solution.

2 2

2 2

10.0 g CaCl 1 mol CaCl 2 mol Cl[Cl ] 0.180

1 L soln 111.0 g CaCl 1 mol CaCl

−−

= × × = M

AgCl(s) ⇌ Ag+(aq) + Cl

−(aq)

Initial (M): 0.000 0.180

Change (M): −s +s +s

Equilibrium (M): s (0.180 + s)

Page 29: Chapter 16 solution manual 11e

CHAPTER 16: ACID-BASE EQUILIBRIA AND SOLUBILITY EQUILIBRIA 570

If we assume that (0.180 + s) ≈ 0.180, then

Ksp = [Ag+][Cl

−] = 1.6 × 10

−10

10

sp 101.6 10[Ag ] 8.9 10

0.180[Cl ]

−+ −

×= = = × =

KM s

The molar solubility of AgCl is 8.9 ×××× 10−−−−10

M.

16.70 (a) The equilibrium reaction is:

BaSO4(s) ⇌ Ba2+

(aq) + 24SO ( )−

aq

Initial (M): 0 0

Change (M): −s +s +s

Equilibrium (M): s s

Ksp = [Ba2+

]24[SO ]−

1.1 × 10−10

= s2

s ==== 1.0 ×××× 10−−−−5

M

The molar solubility of BaSO4 in pure water is 1.0 × 10−5

mol/L.

(b) The initial concentration of24SO −

is 1.0 M.

BaSO4(s) ⇌ Ba2+

(aq) + 24SO ( )−

aq

Initial (M): 0 1.0

Change (M): −s +s +s

Equilibrium (M): s 1.0 + s

Ksp = [Ba2+

]24[SO ]−

1.1 × 10−10

= (s)(1.0 + s) ≈ (s)(1.0)

s ==== 1.1 ×××× 10−−−−10

M

Due to the common ion effect, the molar solubility of BaSO4 decreases to 1.1 × 10−10

mol/L in

1.0 M24SO ( )−

aq compared to 1.0 × 10−5

mol/L in pure water.

16.71 When the anion of a salt is a base, the salt will be more soluble in acidic solution because the hydrogen ion

decreases the concentration of the anion (Le Chatelier's principle):

B−(aq) + H

+(aq) ⇌ HB(aq)

(a) BaSO4 will be slightly more soluble because24SO −

is a base (although a weak one).

(b) The solubility of PbCl2 in acid is unchanged over the solubility in pure water because HCl is a strong

acid, and therefore Cl− is a negligibly weak base.

(c) Fe(OH)3 will be more soluble in acid because OH− is a base.

(d) CaCO3 will be more soluble in acidic solution because the23CO −

ions react with H+ ions to form CO2

and H2O. The CO2 escapes from the solution, shifting the equilibrium. Although it is not important in

this case, the carbonate ion is also a base.

Page 30: Chapter 16 solution manual 11e

CHAPTER 16: ACID-BASE EQUILIBRIA AND SOLUBILITY EQUILIBRIA 571

16.72 (b) 24SO ( )−

aq is a weak base

(c) OH−(aq) is a strong base

(d) 2

2 4C O ( )−aq is a weak base

(e) 34PO ( )−

aq is a weak base.

The solubilities of the above will increase in acidic solution. Only (a), which contains an extremely weak

base (I− is the conjugate base of the strong acid HI) is unaffected by the acid solution.

16.73 In water:

Mg(OH)2 ⇌ Mg2+

+ 2OH−

s 2s

Ksp = 4s3 = 1.2 × 10

−11

s = 1.4 ×××× 10−−−−4

M

In a buffer at pH = 9.0

[H+] = 1.0 × 10

−9

[OH−] = 1.0 × 10

−5

1.2 × 10−11

= (s)(1.0 × 10−5

)2

s = 0.12 M

16.74 From Table 16.2, the value of Ksp for iron(II) is 1.6 × 10

−14.

(a) At pH = 8.00, pOH = 14.00 − 8.00 = 6.00, and [OH−] = 1.0 × 10

−6 M

14

sp2

2 6 2

1.6 10[Fe ] 0.016

[OH ] (1.0 10 )

−+

− −

×= = =

×

KM

The molar solubility of iron(II) hydroxide at pH = 8.00 is 0.016 M

(b) At pH = 10.00, pOH = 14.00 − 10.00 = 4.00, and [OH−] = 1.0 × 10

−4 M

14

sp2 6

2 4 2

1.6 10[Fe ] 1.6 10

[OH ] (1.0 10 )

−+ −

− −

×= = = ×

×

KM

The molar solubility of iron(II) hydroxide at pH = 10.00 is 1.6 ×××× 10−−−−6

M.

16.75 The solubility product expression for magnesium hydroxide is

Ksp = [Mg2+

][OH−]2 = 1.2 × 10

−11

We find the hydroxide ion concentration when [Mg2+

] is 1.0 × 10−10

M

1

11 2

10

1.2 10

1.0 10

×= = ×

[OH ] 0.35−−−− M

Therefore the concentration of OH− must be slightly greater than 0.35 M.

Page 31: Chapter 16 solution manual 11e

CHAPTER 16: ACID-BASE EQUILIBRIA AND SOLUBILITY EQUILIBRIA 572

16.76 We first determine the effect of the added ammonia. Let's calculate the concentration of NH3. This is a

dilution problem.

MiVi = MfVf

(0.60 M)(2.00 mL) = Mf(1002 mL)

Mf = 0.0012 M NH3

Ammonia is a weak base (Kb = 1.8 × 10−5

).

NH3 + H2O ⇌ 4NH+ + OH

Initial (M): 0.0012 0 0

Change (M): −x +x +x

Equil. (M): 0.0012 − x x x

4b

3

[NH ][OH ]

[NH ]

+ −

=K

2

51.8 10(0.0012 )

−× =

x

x

Solving the resulting quadratic equation gives x = 0.00014, or [OH−] = 0.00014 M

This is a solution of iron(II) sulfate, which contains Fe2+

ions. These Fe2+

ions could combine with OH− to

precipitate Fe(OH)2. Therefore, we must use Ksp for iron(II) hydroxide. We compute the value of Qc for this

solution.

Fe(OH)2(s) ⇌ Fe2+

(aq) + 2OH−(aq)

Q = [Fe2+

]0[OH−]0

2 = (1.0 × 10

−3)(0.00014)

2 = 2.0 × 10

−11

Note that when adding 2.00 mL of NH3 to 1.0 L of FeSO4, the concentration of FeSO4 will decrease slightly.

However, rounding off to 2 significant figures, the concentration of 1.0 × 10−3

M does not change. Q is larger

than Ksp [Fe(OH)2] = 1.6 × 10−14

. The concentrations of the ions in solution are greater than the equilibrium

concentrations; the solution is saturated. The system will shift left to reestablish equilibrium; therefore, a

precipitate of Fe(OH)2 will form.

16.79 First find the molarity of the copper(II) ion

41 mol

Moles CuSO 2.50 g 0.0157 mol159.6 g

= × =

2 0.0157 mol

[Cu ] 0.01740.90 L

+= = M

As in Example 16.15 of the text, the position of equilibrium will be far to the right. We assume essentially all

the copper ion is complexed with NH3. The NH3 consumed is 4 × 0.0174 M = 0.0696 M. The uncombined

NH3 remaining is (0.30 − 0.0696) M, or 0.23 M. The equilibrium concentrations of2

3 4Cu(NH ) +and NH3 are

therefore 0.0174 M and 0.23 M, respectively. We find [Cu2+

] from the formation constant expression.

2133 4

f 2 4 2 43

[Cu(NH ) ] 0.01745.0 10

[Cu ][NH ] [Cu ](0.23)

+

+ += = × =K

[Cu2++++

] = 1.2 ×××× 10−−−−13

M

Page 32: Chapter 16 solution manual 11e

CHAPTER 16: ACID-BASE EQUILIBRIA AND SOLUBILITY EQUILIBRIA 573

16.80 Strategy: The addition of Cd(NO3)2 to the NaCN solution results in complex ion formation. In solution,

Cd2+

ions will complex with CN− ions. The concentration of Cd

2+ will be determined by the following

equilibrium

Cd2+

(aq) + 4CN−(aq) ⇌

24Cd(CN) −

From Table 16.4 of the text, we see that the formation constant (Kf) for this reaction is very large

(Kf = 7.1 × 1016

). Because Kf is so large, the reaction lies mostly to the right. At equilibrium, the

concentration of Cd2+

will be very small. As a good approximation, we can assume that essentially all the

dissolved Cd2+

ions end up as24Cd(CN) −

ions. What is the initial concentration of Cd2+

ions? A very small

amount of Cd2+

will be present at equilibrium. Set up the Kf expression for the above equilibrium to solve

for [Cd2+

].

Solution: Calculate the initial concentration of Cd2+

ions.

2+3 2

2 33 2 3 20

1 mol Cd(NO ) 1 mol Cd0.50 g

236.42 g Cd(NO ) 1 mol Cd(NO ) [Cd ] 4.2 10

0.50 L

+ −

× ×

= = × M

If we assume that the above equilibrium goes to completion, we can write

Cd2+

(aq) + 4CN−(aq) →

24Cd(CN) ( )−

aq

Initial (M): 4.2 × 10−3

0.50 0

Change (M): −4.2 × 10−3

−4(4.2 × 10−3

) +4.2 × 10−3

Final (M): 0 0.48 4.2 × 10−3

To find the concentration of free Cd2+

at equilibrium, use the formation constant expression.

24

f 2 4

[Cd(CN) ]

[Cd ][CN ]

+ −=K

Rearranging,

22 4

4f

[Cd(CN) ][Cd ]

[CN ]

−+

−=

K

Substitute the equilibrium concentrations calculated above into the formation constant expression to calculate

the equilibrium concentration of Cd2+

.

2 34

4 16 4f

[Cd(CN) ] 4.2 10

[CN ] (7.1 10 )(0.48)

− −

×= = =

×

2 18[Cd ] 1.1 10

KM

+ −+ −+ −+ −××××

[CN−−−−] = 0.48 M + 4(1.1 × 10

−18 M) = 0.48 M

24[Cd(CN) ]

−−−− = (4.2 × 10−3

M) − (1.1 × 10−18

) = 4.2 ×××× 10−−−−3

M

Check: Substitute the equilibrium concentrations calculated into the formation constant expression to

calculate Kf. Also, the small value of [Cd2+

] at equilibrium, compared to its initial concentration of

4.2 × 10−3

M, certainly justifies our approximation that almost all the Cd2+

ions react.

Page 33: Chapter 16 solution manual 11e

CHAPTER 16: ACID-BASE EQUILIBRIA AND SOLUBILITY EQUILIBRIA 574

16.81 The reaction

Al(OH)3(s) + OH−(aq) ⇌ 4Al(OH) ( )−

aq

is the sum of the two known reactions

Al(OH)3(s) ⇌ Al3+

(aq) + 3OH−(aq) Ksp = 1.8 × 10

−33

Al3+

(aq) + 4OH−(aq) ⇌ 4Al(OH) ( )−

aq Kf = 2.0 × 1033

The equilibrium constant is

33 33 4sp f

[Al(OH) ](1.8 10 )(2.0 10 ) 3.6

[OH ]

−−

−= = × × = =K K K

When pH = 14.00, [OH−] = 1.0 M, therefore

4[Al(OH) ]− = K[OH

−] = (3.6)(1 M) = 3.6 M

This represents the maximum possible concentration of the complex ion at pH 14.00. Since this is much

larger than the initial 0.010 M, the complex ion, ,4Al(OH)−−−− will be the predominant species.

16.82 Silver iodide is only slightly soluble. It dissociates to form a small amount of Ag

+ and I

− ions. The Ag

+ ions

then complex with NH3 in solution to form the complex ion 3 2Ag(NH ) .+ The balanced equations are:

AgI(s) ⇌ Ag+(aq) + I

−(aq) Ksp = [Ag

+][I

−] = 8.3 × 10

−17

Ag+(aq) + 2NH3(aq) ⇌ 3 2Ag(NH ) ( )+

aq 73 2

f 23

[Ag(NH ) ]1.5 10

[Ag ][NH ]

+

+= = ×K

Overall: AgI(s) + 2NH3(aq) ⇌ 3 2Ag(NH ) ( )+aq + I

−(aq) K = Ksp × Kf = 1.2 × 10

−9

If s is the molar solubility of AgI then,

AgI(s) + 2NH3(aq) ⇌ 3 2Ag(NH ) ( )+aq + I

−(aq)

Initial (M): 1.0 0.0 0.0

Change (M): −s −2s +s +s

Equilibrium (M): (1.0 − 2s) s s

Because Kf is large, we can assume all of the silver ions exist as 3 2Ag(NH ) .+ Thus,

3 2[Ag(NH ) ]+ = [I

−] = s

We can write the equilibrium constant expression for the above reaction, then solve for s.

9

2 2

( )( ) ( )( )1.2 10

(1.0 2 ) (1.0)

−= × = ≈

s s s sK

s

s ==== 3.5 ×××× 10−−−−5

M

At equilibrium, 3.5 × 10−5

moles of AgI dissolves in 1 L of 1.0 M NH3 solution.

Page 34: Chapter 16 solution manual 11e

CHAPTER 16: ACID-BASE EQUILIBRIA AND SOLUBILITY EQUILIBRIA 575

16.83 The balanced equations are:

Ag+(aq) + 2NH3(aq) ⇌ 3 2Ag(NH ) ( )+

aq

Zn2+

(aq) + 4NH3(aq) ⇌ 2

3 4Zn(NH ) ( )+aq

Zinc hydroxide forms a complex ion with excess OH− and silver hydroxide does not; therefore, zinc

hydroxide is soluble in 6 M NaOH.

16.84 (a) The equations are as follows:

CuI2(s) ⇌ Cu2+

(aq) + 2I−(aq)

Cu2++++

(aq) ++++ 4NH3(aq) ⇌ 2

3 4Cu(NH ) ( )++++ aq

The ammonia combines with the Cu2+

ions formed in the first step to form the complex ion,

23 4Cu(NH ) ,+

effectively removing the Cu2+

ions, causing the first equilibrium to shift to the right

(resulting in more CuI2 dissolving).

(b) Similar to part (a):

AgBr(s) ⇌ Ag+(aq) + Br

−(aq)

Ag++++(aq) ++++ 2CN

−−−−(aq) ⇌ 2Ag(CN) ( )

−−−− aq

(c) Similar to parts (a) and (b).

HgCl2(s) ⇌ Hg2+

(aq) + 2Cl−(aq)

Hg2++++

(aq) ++++ 4Cl−−−−(aq) ⇌

24HgCl ( )

−−−− aq

16.87 Silver chloride will dissolve in aqueous ammonia (NH3) because of the formation of a complex ion. Lead

chloride will not dissolve; it doesn’t form an ammonia complex.

16.88 Since some PbCl2 precipitates, the solution is saturated. From Table 16.2, the value of Ksp for lead(II)

chloride is 2.4 × 10−4

. The equilibrium is:

PbCl2(aq) ⇌ Pb2+

(aq) + 2Cl−(aq)

We can write the solubility product expression for the equilibrium.

Ksp = [Pb2+

][Cl−]2

Ksp and [Cl−] are known. Solving for the Pb

2+ concentration,

4

sp

2 2

2.4 10

[Cl ] (0.15)

×= = =

2[Pb ] 0.011

KM

++++

16.89 Ammonium chloride is the salt of a weak base (ammonia). It will react with strong aqueous hydroxide to

form ammonia (Le Châtelier’s principle).

NH4Cl(s) + OH−(aq) → NH3(g) + H2O(l) + Cl

−(aq)

The human nose is an excellent ammonia detector. Nothing happens between KCl and strong aqueous NaOH.

Page 35: Chapter 16 solution manual 11e

CHAPTER 16: ACID-BASE EQUILIBRIA AND SOLUBILITY EQUILIBRIA 576

16.90 Chloride ion will precipitate Ag+ but not Cu

2+. So, dissolve some solid in H2O and add HCl. If a precipitate

forms, the salt was AgNO3. A flame test will also work. Cu2+

gives a green flame test.

16.91 (a) According to the Henderson-Hasselbalch equation:

a[conjugate base]

pH p log[acid]

= +K

If: [conjugate base]

10[acid]

= , then:

pH = pKa + log 10 = pKa + 1

If: [conjugate base]

0.1[acid]

= , then:

pH = pKa + log 0.1 = pKa − 1

Therefore, the concentration ratio over which the buffer is effective is:

pH = pKa ± 1

(b) Ka (CH3COOH) = 1.8 × 10−5

, pH = pKa ± 1 = 4.74 ± 1

The pH range is 3.74 to 5.74.

(c) Ka (HNO2) = 4.5 × 10−4

, pH = pKa ± 1 = 3.35 ± 1

The pH range is 2.35 to 4.35.

(d) Ka (H2CO3) = 4.2 × 10−7

, pH = pKa ± 1 = 6.38 ± 1

The pH range is 5.38 to 7.38.

(e) Ka24(HPO )−

= 4.8 × 10−13

, pH = pKa ± 1 = 12.32 ± 1

The pH range is 11.32 to 13.32.

16.92 We can use the Henderson-Hasselbalch equation to solve for the pH when the indicator is 90% acid /

10% conjugate base and when the indicator is 10% acid / 90% conjugate base.

a[conjugate base]

pH p log[acid]

= +K

Solving for the pH with 90% of the indicator in the HIn form:

[10]

pH 3.46 log 3.46 0.95 2.51[90]

= + = − =

Next, solving for the pH with 90% of the indicator in the In− form:

[90]

pH 3.46 log 3.46 0.95 4.41[10]

= + = + =

Thus the pH range varies from 2.51 to 4.41 as the [HIn] varies from 90% to 10%.

Page 36: Chapter 16 solution manual 11e

CHAPTER 16: ACID-BASE EQUILIBRIA AND SOLUBILITY EQUILIBRIA 577

16.93 The mass percent of iodide in the sample is:

mass iodidemass % iodide 100%

mass sample (4.50 g)= ×

The mass of the sample is given (4.50 g). We can determine the mass of iodide in the sample from the moles

of AgNO3 needed for precipitation. The reaction is:

I−(aq) + AgNO3(aq) → AgI(s) + 3NO−

(aq)

3

3

0.186 mol 1 mol I5.54 mL 0.00103 mol AgNO 0.00103 mol I

1000 mL soln 1 mol AgNO

−−

× = × =

126.9 g I0.00103 mol I 0.131 g I

1 mol I

−− −

−× =

The mass percent of iodide in the sample is:

0.131 g100%

4.50 g= × =mass % iodide 2.91%

16.94 The hydrochloric acid reacts with CH3COO

− to produce CH3COOH. Note that the final solution volume is

1.0 L; therefore, the concentration of CH3COO− is 0.010 M.

CH3COO−(aq) + HCl(aq) → CH3COOH(aq) + Cl

−(aq)

Initial (M): 0.010 x 0

Change (M): −x −x +x

Final (M): 0.010 – x 0 x

Next, we use the Henderson-Hasselbalch equation to solve for x.

CH3COOH(aq) ⇌ H+(aq) + CH3COO

−(aq)

Ka = 1.8 × 10−5

, pKa = 4.74

3

a3

[CH COO ]pH p log

[CH COOH]

= +K

0.010

5.00 4.74 log−

= +

x

x

0.26 0.010

10−

=x

x

x = 0.0035 M

Because the mixed solution volume is 1.0 L, the moles of HCl needed is 0.0035 mole. The volume of HCl

needed to prepare this buffer with a pH = 5.00 is:

1 L0.0035 mol HCl

0.020 mol× = = ×

20.18 L 1.8 10 mL

Page 37: Chapter 16 solution manual 11e

CHAPTER 16: ACID-BASE EQUILIBRIA AND SOLUBILITY EQUILIBRIA 578

16.95 Referring to Figure 16.5, at the half-equivalence point, [weak acid] = [conjugate base]. Using the Henderson-

Hasselbalch equation:

a[conjugate base]

pH p log[acid]

= +K

so,

pH = pKa

16.96 First, calculate the pH of the 2.00 M weak acid (HNO2) solution before any NaOH is added.

HNO2(aq) ⇌ H+(aq) + 2NO ( )−

aq

Initial (M): 2.00 0 0

Change (M): −x +x +x

Equilibrium (M): 2.00 − x x x

2a

2

[H ][NO ]

[HNO ]

+ −

=K

2 2

44.5 102.00 2.00

−× = ≈

x x

x

x = [H+] = 0.030 M

pH = −log(0.030) = 1.52

Since the pH after the addition is 1.5 pH units greater, the new pH = 1.52 + 1.50 = 3.02.

From this new pH, we can calculate the [H+] in solution.

[H+] = 10

−pH = 10

−3.02 = 9.55 × 10

−4 M

When the NaOH is added, we dilute our original 2.00 M HNO2 solution to:

MiVi = MfVf

(2.00 M)(400 mL) = Mf(600 mL)

Mf = 1.33 M

Since we have not reached the equivalence point, we have a buffer solution. The reaction between HNO2 and

NaOH is:

HNO2(aq) + NaOH(aq) → NaNO2(aq) + H2O(l)

Since the mole ratio between HNO2 and NaOH is 1:1, the decrease in [HNO2] is the same as the decrease in

[NaOH].

We can calculate the decrease in [HNO2] by setting up the weak acid equilibrium. From the pH of the

solution, we know that the [H+] at equilibrium is 9.55 × 10

−4 M.

HNO2(aq) ⇌ H+(aq) + 2NO ( )−

aq

Initial (M): 1.33 0 0

Change (M): −x +x

Equilibrium (M): 1.33 − x 9.55 × 10−4

x

Page 38: Chapter 16 solution manual 11e

CHAPTER 16: ACID-BASE EQUILIBRIA AND SOLUBILITY EQUILIBRIA 579

We can calculate x from the equilibrium constant expression.

2a

2

[H ][NO ]

[HNO ]

+ −

=K

4

4 (9.55 10 )( )4.5 10

1.33

−− ×

× =−

x

x

x = 0.426 M

Thus, x is the decrease in [HNO2] which equals the concentration of added OH−. However, this is the

concentration of NaOH after it has been diluted to 600 mL. We need to correct for the dilution from

200 mL to 600 mL to calculate the concentration of the original NaOH solution.

MiVi = MfVf

Mi(200 mL) = (0.426 M)(600 mL)

[NaOH] = Mi = 1.28 M

16.97 The Ka of butyric acid is obtained by taking the antilog of 4.7 (10

−4.7) which is 2 × 10

−5. The value of Kb is:

14w

5a

1.0 10

2 10

−−

×= = = ×

×

10b 5 10

K

KK

16.98 The resulting solution is not a buffer system. There is excess NaOH and the neutralization is well past the

equivalence point.

0.167 mol

Moles NaOH 0.500 L 0.0835 mol1 L

= × =

0.100 mol

Moles HCOOH 0.500 L 0.0500 mol1 L

= × =

HCOOH(aq) + NaOH(aq) → HCOONa(aq) + H2O(l)

Initial (mol): 0.0500 0.0835 0

Change (mol): −0.0500 −0.0500 +0.0500

Final (mol): 0 0.0335 0.0500

The volume of the resulting solution is 1.00 L (500 mL + 500 mL = 1000 mL).

0.0335 mol

1.00 L= =[OH ] 0.0335 M

−−−−

(0.0335 0.0500) mol

1.00 L

+= =[Na ] 0.0835++++ M

14

w 1.0 10

0.0335[OH ]

×= = =

13[H ] 3.0 10

KM

+ −+ −+ −+ −××××

0.0500 mol

1.00 L= =[HCOO ] 0.0500−−−− M

HCOO−(aq) + H2O(l) ⇌ HCOOH(aq) + OH

−(aq)

Initial (M): 0.0500 0 0.0335

Change (M): −x +x +x

Equilibrium (M): 0.0500 − x x 0.0335 + x

Page 39: Chapter 16 solution manual 11e

CHAPTER 16: ACID-BASE EQUILIBRIA AND SOLUBILITY EQUILIBRIA 580

b[HCOOH][OH ]

[HCOO ]

−=K

11 ( )(0.0335 ) ( )(0.0335)

5.9 10(0.0500 ) (0.0500)

− +× = ≈

x x x

x

x = [HCOOH] = 8.8 ×××× 10−−−−11

M

16.99 Most likely the increase in solubility is due to complex ion formation:

Cd(OH)2(s) + 2OH− ⇌

24Cd(OH) ( )−

aq

This is a Lewis acid-base reaction.

16.100 The number of moles of Ba(OH)2 present in the original 50.0 mL of solution is:

22

1.00 mol Ba(OH)50.0 mL 0.0500 mol Ba(OH)

1000 mL soln× =

The number of moles of H2SO4 present in the original 86.4 mL of solution, assuming complete dissociation,

is:

2 42 4

0.494 mol H SO86.4 mL 0.0427 mol H SO

1000 mL soln× =

The reaction is:

Ba(OH)2(aq) + H2SO4(aq) → BaSO4(s) + 2H2O(l)

Initial (mol): 0.0500 0.0427 0

Change (mol): −0.0427 −0.0427 +0.0427

Final (mol): 0.0073 0 0.0427

Thus the mass of BaSO4 formed is:

44

4

233.4 g BaSO0.0427 mol BaSO

1 mol BaSO× = 49.97 g BaSO

The pH can be calculated from the excess OH− in solution. First, calculate the molar concentration of OH

−.

The total volume of solution is 136.4 mL = 0.1364 L.

22

2 mol OH0.0073 mol Ba(OH)

1 mol Ba(OH)[OH ] 0.11

0.1364 L

×

= = M

pOH = −log(0.11) = 0.96

pH = 14.00 − pOH = 14.00 − 0.96 = 13.04

16.101 A solubility equilibrium is an equilibrium between a solid (reactant) and its components (products: ions,

neutral molecules, etc.) in solution. Only (d) represents a solubility equilibrium.

Consider part (b). Can you write the equilibrium constant for this reaction in terms of Ksp for calcium

phosphate?

Page 40: Chapter 16 solution manual 11e

CHAPTER 16: ACID-BASE EQUILIBRIA AND SOLUBILITY EQUILIBRIA 581

16.102 First, we calculate the molar solubility of CaCO3.

CaCO3(s) ⇌ Ca2+

(aq) + 23CO ( )−

aq

Initial (M): 0 0

Change (M): −s +s +s

Equil. (M): s s

Ksp = [Ca2+

]23[CO ]−

= s2 = 8.7 × 10

−9

s = 9.3 × 10−5

M = 9.3 × 10−5

mol/L

The moles of CaCO3 in the kettle are:

33

3

1 mol CaCO116 g 1.16 mol CaCO

100.1 g CaCO× =

The volume of distilled water needed to dissolve 1.16 moles of CaCO3 is:

43 5

3

1 L1.16 mol CaCO 1.2 10 L

9.3 10 mol CaCO−× = ×

×

The number of times the kettle would have to be filled is:

4 1 filling

(1.2 10 L)2.0 L

× × =36.0 10 fillings××××

Note that the very important assumption is made that each time the kettle is filled, the calcium carbonate is

allowed to reach equilibrium before the kettle is emptied.

16.103 Since equal volumes of the two solutions were used, the initial molar concentrations will be halved.

0.12

[Ag ] 0.0602

+= =

MM

2(0.14 )

[Cl ] 0.142

−= =

MM

Let’s assume that the Ag+ ions and Cl

− ions react completely to form AgCl(s). Then, we will reestablish the

equilibrium between AgCl, Ag+, and Cl

−.

Ag+(aq) + Cl

−(aq) → AgCl(s)

Initial (M): 0.060 0.14 0

Change (M): −0.060 −0.060 +0.060

Final (M): 0 0.080 0.060

Now, setting up the equilibrium,

AgCl(s) ⇌ Ag+(aq) + Cl

−(aq)

Initial (M): 0.060 0 0.080

Change (M): −s +s +s

Equilibrium (M): 0.060 − s s 0.080 + s

Page 41: Chapter 16 solution manual 11e

CHAPTER 16: ACID-BASE EQUILIBRIA AND SOLUBILITY EQUILIBRIA 582

Set up the Ksp expression to solve for s.

Ksp = [Ag+][Cl

−]

1.6 × 10−10

= (s)(0.080 + s)

s = 2.0 × 10−9

M

[Ag++++] = s = 2.0 ×××× 10

−−−−9 M

[Cl−−−−] = 0.080 M + s = 0.080 M

0.14

2= =

2[Zn ] 0.070

M++++Μ

0.12

2= =3[NO ] 0.060

MM

−−−−

16.104 First we find the molar solubility and then convert moles to grams. The solubility equilibrium for silver

carbonate is:

Ag2CO3(s) ⇌ 2Ag+(aq) +

23CO ( )−

aq

Initial (M): 0 0

Change (M): −s +2s +s

Equilibrium (M): 2s s

Ksp = [Ag+]2 2

3[CO ]− = (2s)

2(s) = 4s

3 = 8.1 × 10

−12

1

12 348.1 10

1.3 104

−−

×= = ×

s M

Converting from mol/L to g/L:

41.3 10 mol 275.8 g

1 L soln 1 mol

−×

× = 0.036 g/L

16.105 For Mg(OH)2, Ksp = 1.2 × 10

−11. When [Mg

2+] = 0.010 M, the [OH

−] value is

Ksp = [Mg2+

][OH−]2

or

1

2sp

2[OH ]

[Mg ]

+

=

K

1

11 251.2 10

[OH ] 3.5 100.010

−− −

×= = ×

M

This [OH−] corresponds to a pH of 9.54. In other words, Mg(OH)2 will begin to precipitate from this solution

at pH of 9.54.

For Zn(OH)2, Ksp = 1.8 × 10−14

. When [Zn2+

] = 0.010 M, the [OH−] value is

1

2sp

2[OH ]

[Zn ]

+

=

K

Page 42: Chapter 16 solution manual 11e

CHAPTER 16: ACID-BASE EQUILIBRIA AND SOLUBILITY EQUILIBRIA 583

1

14 261.8 10

[OH ] 1.3 100.010

−− −

×= = ×

M

This corresponds to a pH of 8.11. In other words Zn(OH)2 will begin to precipitate from the solution at

pH = 8.11. These results show that Zn(OH)2 will precipitate when the pH just exceeds 8.11 and that

Mg(OH)2 will precipitate when the pH just exceeds 9.54. Therefore, to selectively remove zinc as Zn(OH)2,

the pH must be greater than 8.11 but less than 9.54.

16.106 (a) To 2.50 × 10

−3 mole HCl (that is, 0.0250 L of 0.100 M solution) is added 1.00 × 10

−3 mole CH3NH2

(that is, 0.0100 L of 0.100 M solution).

HCl(aq) + CH3NH2(aq) → CH3NH3Cl(aq)

Initial (mol): 2.50 × 10−3

1.00 × 10−3

0

Change (mol): −1.00 × 10−3

−1.00 × 10−3

+1.00 × 10−3

Equilibrium (mol): 1.50 × 10−3

0 1.00 × 10−3

After the acid-base reaction, we have 1.50 × 10−3

mol of HCl remaining. Since HCl is a strong acid, the

[H+] will come from the HCl. The total solution volume is 35.0 mL = 0.0350 L.

31.50 10 mol

[H ] 0.04290.0350 L

−+ ×

= = M

pH ==== 1.37

(b) When a total of 25.0 mL of CH3NH2 is added, we reach the equivalence point. That is, 2.50 × 10−3

mol

HCl reacts with 2.50 × 10−3

mol CH3NH2 to form 2.50 × 10−3

mol CH3NH3Cl. Since there is a total of

50.0 mL of solution, the concentration of CH3NH3+ is:

3

23 3

2.50 10 mol[CH NH ] 5.00 10

0.0500 L

−+ −×

= = × M

This is a problem involving the hydrolysis of the weak acid CH3NH3+.

3 3CH NH ( )+aq ⇌ H

+(aq) + CH3NH2(aq)

Initial (M): 5.00 × 10−2

0 0

Change (M): −x +x +x

Equilibrium (M): (5.00 × 10−2

) − x x x

3 2a

3 3

[CH NH ][H ]

[CH NH ]

+

+=K

2 2

11

2 22.3 10

(5.00 10 ) 5.00 10

− −× = ≈

× − ×

x x

x

1.15 × 10−12

= x2

x = 1.07 × 10−6

M = [H+]

pH ==== 5.97

Page 43: Chapter 16 solution manual 11e

CHAPTER 16: ACID-BASE EQUILIBRIA AND SOLUBILITY EQUILIBRIA 584

(c) 35.0 mL of 0.100 M CH3NH2 (3.50 × 10−3

mol) is added to the 25 mL of 0.100 M HCl (2.50 × 10−3

mol).

HCl(aq) + CH3NH2(aq) → CH3NH3Cl(aq)

Initial (mol): 2.50 × 10−3

3.50 × 10−3

0

Change (mol): −2.50 × 10−3

−2.50 × 10−3

+2.50 × 10−3

Equilibrium (mol): 0 1.00 × 10−3

2.50 × 10−3

This is a buffer solution. Using the Henderson-Hasselbalch equation:

a[conjugate base]

pH p log[acid]

= +K

3

11

3

(1.00 10 )log(2.3 10 ) log

(2.50 10 )

−−

×= − × + =

×pH 10.24

16.107 The equilibrium reaction is:

Pb(IO3)2(aq) ⇌ Pb2+

(aq) + 32IO ( )−aq

Initial (M): 0 0.10

Change (M): −2.4 × 10−11

+2.4 × 10−11

+2(2.4 × 10−11

)

Equilibrium (M): 2.4 × 10−11

≈ 0.10

Substitute the equilibrium concentrations into the solubility product expression to calculate Ksp.

Ksp = [Pb2+

]2

3[IO ]−

Ksp = (2.4 × 10−11

)(0.10)2 = 2.4 ×××× 10

−−−−13

16.108 The precipitate is HgI2.

Hg2+

(aq) + 2I−(aq) → HgI2(s)

With further addition of I−, a soluble complex ion is formed and the precipitate redissolves.

HgI2(s) + 2I−(aq) →

24HgI ( )−

aq

16.109 BaSO4(s) ⇌ Ba2+

(aq) +24SO ( )−

aq

Ksp = [Ba2+

]24[SO ]−

= 1.1 × 10−10

[Ba2+

] = 1.0 × 10−5

M

In 5.0 L, the number of moles of Ba2+

is

(5.0 L)(1.0 × 10−5

mol/L) = 5.0 × 10−5

mol Ba2+

= 5.0 × 10−5

mol BaSO4

The number of grams of BaSO4 dissolved is

5 4

4 44

233.4 g BaSO(5.0 10 mol BaSO ) 0.012 g BaSO

1 mol BaSO

−× × =

In practice, even less BaSO4 will dissolve because the BaSO4 is not in contact with the entire volume of

blood. Ba(NO3)2 is too soluble to be used for this purpose.

Page 44: Chapter 16 solution manual 11e

CHAPTER 16: ACID-BASE EQUILIBRIA AND SOLUBILITY EQUILIBRIA 585

16.110 We can use the Henderson-Hasselbalch equation to solve for the pH when the indicator is 95% acid /

5% conjugate base and when the indicator is 5% acid / 95% conjugate base.

a[conjugate base]

pH p log[acid]

= +K

Solving for the pH with 95% of the indicator in the HIn form:

[5]

pH 9.10 log 9.10 1.28 7.82[95]

= + = − =

Next, solving for the pH with 95% of the indicator in the In− form:

[95]

pH 9.10 log 9.10 1.28 10.38[5]

= + = + =

Thus the pH range varies from 7.82 to 10.38 as the [HIn] varies from 95% to 5%.

16.111 (a) The solubility product expressions for both substances have exactly the same mathematical form and are

therefore directly comparable. The substance having the smaller Ksp (AgBr) will precipitate first. (Why?)

(b) When CuBr just begins to precipitate the solubility product expression will just equal Ksp (saturated

solution). The concentration of Cu+ at this point is 0.010 M (given in the problem), so the concentration

of bromide ion must be:

Ksp = [Cu+][Br

−] = (0.010)[Br

−] = 4.2 × 10

−8

8

64.2 10[Br ] 4.2 10

0.010

−− −×

= = × M

Using this value of [Br−], we find the silver ion concentration

13

sp

6

7.7 10

[Br ] 4.2 10

− −

×= = =

×

7[Ag ] 1.8 10

K+ −+ −+ −+ −

×××× M

(c) The percent of silver ion remaining in solution is:

71.8 10

100% or0.010

−×

= × =3% Ag ( ) 0.0018% 1.8 10 %

M

M

+ −+ −+ −+ −××××aq

Is this an effective way to separate silver from copper?

16.112 (a) We abbreviate the name of cacodylic acid to CacH. We set up the usual table.

CacH(aq) ⇌ Cac−(aq) + H

+(aq)

Initial (M): 0.10 0 0

Change (M): −x +x +x

Equilibrium (M): 0.10 − x x x

Page 45: Chapter 16 solution manual 11e

CHAPTER 16: ACID-BASE EQUILIBRIA AND SOLUBILITY EQUILIBRIA 586

[ ]

a[H ][Cac ]

CacH

+ −

=K

2 2

76.4 100.10 0.10

−× = ≈

x x

x

x = 2.5 × 10−4

M = [H+]

pH = −log(2.5 × 10−4

) = 3.60

(b) We set up a table for the hydrolysis of the anion:

Cac−(aq) + H2O(l) ⇌ CacH(aq) + OH

−(aq)

Initial (M): 0.15 0 0

Change (M): −x +x +x

Equilibrium (M): 0.15 − x x x

The ionization constant, Kb, for Cac− is:

14

8wb 7

a

1.0 101.6 10

6.4 10

−−

×= = = ×

×

KK

K

b[CacH][OH ]

[Cac ]

−=K

2 2

81.6 100.15 0.15

−× = ≈

x x

x

x = 4.9 × 10−5

M

pOH = −log(4.9 × 10−5

) = 4.31

pH = 14.00 − 4.31 = 9.69

(c) Number of moles of CacH from (a) is:

30.10 mol CacH

50.0 mL CacH 5.0 10 mol CacH1000 mL

−× = ×

Number of moles of Cac− from (b) is:

30.15 mol CacNa

25.0 mL CacNa 3.8 10 mol CacNa1000 mL

−× = ×

At this point we have a buffer solution.

3

7a 3

[Cac ] 3.8 10pK log log(6.4 10 ) log

[CacH] 5.0 10

− −−

×= + = − × + =

×pH 6.07

16.113 The initial number of moles of Ag

+ is

40.010 mol Agmol Ag 50.0 mL 5.0 10 mol Ag

1000 mL soln

++ − +

= × = ×

Page 46: Chapter 16 solution manual 11e

CHAPTER 16: ACID-BASE EQUILIBRIA AND SOLUBILITY EQUILIBRIA 587

We can use the counts of radioactivity as being proportional to concentration. Thus, we can use the ratio to

determine the quantity of Ag+ still in solution. However, since our original 50 mL of solution has been

diluted to 500 mL, the counts per mL will be reduced by ten. Our diluted solution would then produce

7402.5 counts per minute if no removal of Ag+ had occurred.

The number of moles of Ag+ that correspond to 44.4 counts are:

4

65.0 10 mol Ag44.4 counts 3.0 10 mol Ag

7402.5 counts

− +− +×

× = ×

33

3

0.030 mol IOOriginal mol of IO 100 mL 3.0 10 mol

1000 mL soln

−− −

= × = ×

The quantity of 3IO−remaining after reaction with Ag

+:

(original moles − moles reacted with Ag+) = (3.0 × 10

−3 mol) − [(5.0 × 10

−4 mol) − (3.0 × 10

−6 mol)]

= 2.5 × 10−3

mol 3IO−

The total final volume is 500 mL or 0.50 L.

6

63.0 10 mol Ag[Ag ] 6.0 10

0.50 L

− ++ −×

= = × M

333

3

2.5 10 mol IO[IO ] 5.0 10

0.50 L

− −− −×

= = × M

AgIO3(s) ⇌ Ag+(aq) + 3IO ( )−

aq

Ksp = [Ag+] 3[IO ]−

= (6.0 × 10−6

)(5.0 × 10−3

) = 3.0 ×××× 10−−−−8

16.114 (a) MCO3 + 2HCl → MCl2 + H2O + CO2

HCl + NaOH → NaCl + H2O

(b) We are given the mass of the metal carbonate, so we need to find moles of the metal carbonate to

calculate its molar mass. We can find moles of MCO3 from the moles of HCl reacted.

Moles of HCl reacted with MCO3 = Total moles of HCl − Moles of excess HCl

5.653 mol

Total moles of HCl 18.68 mL 0.1056 mol HCl1000 mL soln

= × =

1.789 mol

Moles of excess HCl 12.06 mL 0.02158mol1000 mL soln

= × = HCl

Moles of HCl reacted with MCO3 = 0.1056 mol − 0.02158 mol = 0.0840 mol HCl

33 3

1 mol MCOMoles of MCO reacted 0.0840 mol HCl 0.0420 mol MCO

2 mol HCl= × =

3.542 g

0.0420 mol= =3Molar mass of MCO 84.3 g/mol

Page 47: Chapter 16 solution manual 11e

CHAPTER 16: ACID-BASE EQUILIBRIA AND SOLUBILITY EQUILIBRIA 588

Molar mass of CO3 = 60.01 g

Molar mass of M = 84.3 g/mol − 60.01 g/mol = 24.3 g/mol

The metal, M, is Mg! 16.115 (a) H

+ + OH

− → H2O K = 1.0 ×××× 10

14

(b) H+ + NH3 → 4NH+

10

a

1 1

5.6 10−= = =

×

91.8 10K

K ××××

(c) CH3COOH + OH− → CH3COO

− + H2O

Broken into 2 equations:

CH3COOH → CH3COO− + H

+ Ka

H+ + OH

− → H2O 1/Kw

5

a

14w

1.8 10

1.0 10

×= = =

×

91.8 10

K

KK ××××

(d) CH3COOH + NH3 → CH3COONH4

Broken into 2 equations:

CH3COOH → CH3COO− + H

+ Ka

NH3 + H+ → 4NH+

'a

1

K

5

a

' 10a

1.8 10

5.6 10

×= = =

×

43.2 10

K

KK ××××

16.116 The number of moles of NaOH reacted is:

30.500 mol NaOH

15.9 mL NaOH 7.95 10 mol NaOH1000 mL soln

−× = ×

Since two moles of NaOH combine with one mole of oxalic acid, the number of moles of oxalic acid reacted

is 3.98 × 10−3

mol. This is the number of moles of oxalic acid hydrate in 25.0 mL of solution. In 250 mL,

the number of moles present is 3.98 × 10−2

mol. Thus the molar mass is:

2

5.00 g126 g/mol

3.98 10 mol−=

×

From the molecular formula we can write:

2(1.008)g + 2(12.01)g + 4(16.00)g + x(18.02)g = 126 g

Solving for x:

x ==== 2

Page 48: Chapter 16 solution manual 11e

CHAPTER 16: ACID-BASE EQUILIBRIA AND SOLUBILITY EQUILIBRIA 589

16.117 (a) Mix 500 mL of 0.40 M CH3COOH with 500 mL of 0.40 M CH3COONa. Since the final volume is

1.00 L, then the concentrations of the two solutions that were mixed must be one-half of their initial

concentrations.

(b) Mix 500 mL of 0.80 M CH3COOH with 500 mL of 0.40 M NaOH. (Note: half of the acid reacts with

all of the base to make a solution identical to that in part (a) above.)

CH3COOH + NaOH → CH3COONa + H2O

(c) Mix 500 mL of 0.80 M CH3COONa with 500 mL of 0.40 M HCl. (Note: half of the salt reacts with all

of the acid to make a solution identical to that in part (a) above.)

CH3COO− + H

+ → CH3COOH

16.118 (a) a[conjugate base]

pH p log[acid]

= +K

[ionized]

8.00 9.10 log[un-ionized]

= +

[un-ionized]

[ionized]= 12.6 (1)

(b) First, let's calculate the total concentration of the indicator. 2 drops of the indicator are added and each

drop is 0.050 mL.

0.050 mL phenolphthalein

2 drops 0.10 mL phenolphthalein1 drop

× =

This 0.10 mL of phenolphthalein of concentration 0.060 M is diluted to 50.0 mL.

MiVi = MfVf

(0.060 M)(0.10 mL) = Mf(50.0 mL)

Mf = 1.2 × 10−4

M

Using equation (1) above and letting y = [ionized], then [un-ionized] = (1.2 × 10−4

) − y.

4(1.2 10 )

12.6−

× −=

y

y

y ==== 8.8 ×××× 10−−−−6

M

16.119 The sulfur-containing air-pollutants (like H2S) reacts with Pb

2+ to form PbS, which gives paintings a

darkened look.

16.120 (a) Add sulfate. Na2SO4 is soluble, BaSO4 is not.

(b) Add sulfide. K2S is soluble, PbS is not

(c) Add iodide. ZnI2 is soluble, HgI2 is not.

Page 49: Chapter 16 solution manual 11e

CHAPTER 16: ACID-BASE EQUILIBRIA AND SOLUBILITY EQUILIBRIA 590

16.121 Strontium sulfate is the more soluble of the two compounds. Therefore, we can assume that all of the24SO −

ions come from SrSO4.

SrSO4(s) ⇌ Sr2+

(aq) +24SO ( )−

aq

Ksp = [Sr2+

]24[SO ]−

= s2 = 3.8 × 10

−7

7

3.8 10−

= = = × =2 2 4

4[Sr ] [SO ] 6.2 10s+ − −+ − −+ − −+ − −

×××× M

For BaSO4:

10

sp

2 44

1.1 10

[SO ] 6.2 10

−−

− −

×= = =

×

2 7[Ba ] 1.8 10

K++++

×××× M

16.122 The amphoteric oxides cannot be used to prepare buffer solutions because they are insoluble in water.

16.123 CaSO4 ⇌ Ca2+

+24SO −

s2 = 2.4 × 10

−5

s = 4.9 × 10−3

M

34.9 10 mol 136.2 g

1 L 1 mol

−×

= × =Solubility 0.67 g/L

Ag2SO4 ⇌ 2Ag+ +

24SO −

2s s

1.4 × 10−5

= 4s3

s = 0.015 M

0.015 mol 311.1 g

1 L 1 mol= × =Solubility 4.7 g/L

Note: Ag2SO4 has a larger solubility.

16.124 The ionized polyphenols have a dark color. In the presence of citric acid from lemon juice, the anions are

converted to the lighter-colored acids.

16.125 2 4H PO− ⇌ H

+ +

24HPO −

Ka = 6.2 × 10−8

pKa = 7.20

24

2 4

[HPO ]7.50 7.20 log

[H PO ]

−= +

24

2 4

[HPO ]2.0

[H PO ]

−=

Page 50: Chapter 16 solution manual 11e

CHAPTER 16: ACID-BASE EQUILIBRIA AND SOLUBILITY EQUILIBRIA 591

We need to add enough NaOH so that

24 2 4[HPO ] 2[H PO ]− −

=

Initially there was

0.200 L × 0.10 mol/L = 0.020 mol NaH2PO4 present.

For24 2 4[HPO ] 2[H PO ],− −

= we must add enough NaOH to react with 2/3 of the 2 4H PO .− After reaction with

NaOH, we have:

2 4 2 40.020

mol H PO 0.0067 mol H PO3

− −=

mol24HPO −

= 2 × 0.0067 mol = 0.013 mol24HPO −

The moles of NaOH reacted is equal to the moles of24HPO −

produced because the mole ratio between OH−

and24HPO −

is 1:1.

OH− + 2 4H PO−

→ 24HPO −

+ H2O

NaOH

NaOH

mol 0.013 mol0.013 L

1.0 mol/L= = = =NaOH 13 mL

MV

16.126 Assuming the density of water to be 1.00 g/mL, 0.05 g Pb

2+ per 10

6 g water is equivalent to 5 × 10

−5 g Pb

2+/L

2

5 22 2

62 22

1 g H O 1000 mL H O0.05 g Pb5 10 g Pb /L

1 mL H O 1 L H O1 10 g H O

+− +

× × = ××

PbSO4 ⇌ Pb2+

+ 24SO −

Initial (M): 0 0

Change (M): −s +s +s

Equilibrium (M): s s

Ksp = [Pb2+

]24[SO ]−

1.6 × 10−8

= s2

s = 1.3 × 10−4

M

The solubility of PbSO4 in g/L is:

4

21.3 10 mol 303.3 g4.0 10 g/L

1 L 1 mol

−−×

× = ×

Yes. The [Pb2+

] exceeds the safety limit of 5 × 10−5

g Pb2+

/L.

16.127 (a) The acidic hydrogen is from the carboxyl group.

C

O

OH

Page 51: Chapter 16 solution manual 11e

CHAPTER 16: ACID-BASE EQUILIBRIA AND SOLUBILITY EQUILIBRIA 592

(b) At pH 6.50, Equation (16.4) of the text can be written as:

3 [P ]6.50 log(1.64 10 ) log

[HP]

−−

= − × +

3[P ]5.2 10

[HP]

= ×

Thus, nearly all of the penicillin G will be in the ionized form. The ionized form is more soluble in

water because it bears a net charge; penicillin G is largely nonpolar and therefore much less soluble in

water. (Both penicillin G and its salt are effective antibiotics.)

(c) First, the dissolved NaP salt completely dissociates in water as follows:

NaP 2H O→ Na

+ + P

0.12 M 0.12 M

We need to concentrate only on the hydrolysis of the P− ion.

Step 1: Let x be the equilibrium concentrations of HP and OH− due to the hydrolysis of P

− ions. We

summarize the changes:

P−(aq) + H2O(l) ⇌ HP(aq) + OH

−(aq)

Initial (M): 0.12 0 0

Change (M): −x +x +x

Equilibrium (M): 0.12 − x x x

Step 2: 14

12wb 3

a

1.00 106.10 10

1.64 10

−−

×= = = ×

×

KK

K

b[HP][OH ]

[P ]

−=K

2

126.10 100.12

−× =

x

x

Assuming that 0.12 − x ≈ 0.12, we write:

2

126.10 10

0.12

−× =

x

x = 8.6 × 10−7

M

Step 3: At equilibrium:

[OH−] = 8.6 × 10

−7 M

pOH = −log(8.6 × 10−7

) = 6.07

pH = 14.00 − 6.07 = 7.93

Because HP is a relatively strong acid, P− is a weak base. Consequently, only a small fraction of P

undergoes hydrolysis and the solution is slightly basic.

16.128 (c) has the highest [H

+]

F− + SbF5 → 6SbF−

Removal of F− promotes further ionization of HF.

Page 52: Chapter 16 solution manual 11e

CHAPTER 16: ACID-BASE EQUILIBRIA AND SOLUBILITY EQUILIBRIA 593

16.129

0

0.25

0.5

0.75

1

0 1 2 3 4 5 6 7 8 9 10 11 12 13 14

pH

Fra

cti

on

of

sp

ec

ies

pre

se

nt

16.130 (a) This is a common ion23(CO )−

problem.

The dissociation of Na2CO3 is:

Na2CO3(s) 2H O→ 2Na

+(aq) +

23CO ( )−

aq

2(0.050 M) 0.050 M

Let s be the molar solubility of CaCO3 in Na2CO3 solution. We summarize the changes as:

CaCO3(s) ⇌ Ca2+

(aq) + 23CO ( )−

aq

Initial (M): 0.00 0.050

Change (M): +s +s

Equil. (M): +s 0.050 + s

Ksp = [Ca2+

]23[CO ]−

8.7 × 10−9

= s(0.050 + s)

Since s is small, we can assume that 0.050 + s ≈ 0.050

8.7 × 10−9

= 0.050s

s ==== 1.7 ×××× 10−−−−7

M

Thus, the addition of washing soda to permanent hard water removes most of the Ca2+

ions as a result

of the common ion effect.

(b) Mg2+

is not removed by this procedure, because MgCO3 is fairly soluble (Ksp = 4.0 × 10−5

).

(c) The Ksp for Ca(OH)2 is 8.0 × 10−6

.

Ca(OH)2 ⇌ Ca2+

+ 2OH−

At equil.: s 2s

Ksp = 8.0 × 10−6

= [Ca2+

][OH−]2

4s3 = 8.0 × 10

−6

s = 0.0126 M

pH = pKa = 4.74

CH3COOH CH3COO−

Page 53: Chapter 16 solution manual 11e

CHAPTER 16: ACID-BASE EQUILIBRIA AND SOLUBILITY EQUILIBRIA 594

[OH−] = 2s = 0.0252 M

pOH = −log(0.0252) = 1.60

pH ==== 12.40

(d) The [OH−] calculated above is 0.0252 M. At this rather high concentration of OH

−, most of the Mg

2+

will be removed as Mg(OH)2. The small amount of Mg2+

remaining in solution is due to the following

equilibrium:

Mg(OH)2(s) ⇌ Mg2+

(aq) + 2OH−(aq)

Ksp = [Mg2+

][OH−]2

1.2 × 10−11

= [Mg2+

](0.0252)2

[Mg2++++

] ==== 1.9 ×××× 10−−−−8

M

(e) Remove Ca2+

first because it is present in larger amounts.

16.131 a[In ]

pH p log[HIn]

= +K

For acid color:

a1

pH p log10

= +K

pH = pKa − log 10

pH = pKa − 1

For base color:

a10

pH p log1

= +K

pH = pKa + 1

Combining these two equations:

pH ==== pKa ±±±± 1

16.132 pH = pKa + log[ ]

[ ]

conjugate base

acid

At pH ==== 1.0,

−COOH 1.0 = 2.3 + log[ ]

[ ]

−COO

COOH

[ ]

[ ]

−−

COOH

COO = 20

Page 54: Chapter 16 solution manual 11e

CHAPTER 16: ACID-BASE EQUILIBRIA AND SOLUBILITY EQUILIBRIA 595

−NH3+

1.0 = 9.6 + log[ ]

[ ]

−+

NH

NH

2

3

[ ]

[ ]

+NH

NH

3

2

= 4 × 108

Therefore the predominant species is: ++++NH3 −−−− CH2 −−−− COOH

At pH ==== 7.0,

−COOH 7.0 = 2.3 + log[ ]

[ ]

−COO

COOH

[ ]

[ ]

−COO

COOH = 5 × 10

4

−NH3+ 7.0 = 9.6 + log

[ ]

[ ]

−+

NH

NH

2

3

[ ]

[ ]

+NH

NH

3

2

= 4 × 102

Predominant species: ++++NH3 −−−− CH2 −−−− COO

−−−−

At pH ==== 12.0,

−COOH 12.0 = 2.3 + log[ ]

[ ]

−COO

COOH

[ ]

[ ]

−COO

COOH = 5 × 10

9

−NH3+ 12.0 = 9.6 + log

[ ]

[ ]

−+

NH

NH

2

3

[ ]

[ ]

−+

NH

NH

2

3

= 2.5 × 102

Predominant species: NH2 −−−− CH2 −−−− COO−−−−

16.133 (a) The pKb value can be determined at the half-equivalence point of the titration (half the volume of added

acid needed to reach the equivalence point). At this point in the titration pH = pKa, where Ka refers to the

acid ionization constant of the conjugate acid of the weak base. The Henderson-Hasselbalch equation

reduces to pH = pKa when [acid] = [conjugate base]. Once the pKa value is determined, the pKb value can

be calculated as follows:

pKa + pKb = 14.00

(b) Let B represent the base, and BH+ represents its conjugate acid.

B(aq) + H2O(l) ⇌ BH+(aq) + OH

−(aq)

b[BH ][OH ]

[B]

+ −

=K

Page 55: Chapter 16 solution manual 11e

CHAPTER 16: ACID-BASE EQUILIBRIA AND SOLUBILITY EQUILIBRIA 596

b[B][OH ]

[BH ]

+=

K

Taking the negative logarithm of both sides of the equation gives:

b[B]

log[OH ] log log[BH ]

+− = − −K

= b[BH ]

pOH p log[B]

++++

++++K

The titration curve would look very much like Figure 16.5 of the text, except the y-axis would be pOH

and the x-axis would be volume of strong acid added. The pKb value can be determined at the half-

equivalence point of the titration (half the volume of added acid needed to reach the equivalence point).

At this point in the titration, the concentrations of the buffer components, [B] and [BH+], are equal, and

hence pOH = pKb.

16.134 (a) Let x equal the moles of NaOH added to the solution. The acid-base reaction is:

HF(aq) + NaOH(aq) → NaF(aq) + H2O(l)

Initial (mol): 0.0050 x 0

Change (mol): −x −x +x

Final (mol) 0.0050 − x 0 x

At pH = 2.85, we are in the buffer region of the titration. The pH of a 0.20 M HF solution is 1.92, and

clearly the equivalence point of the titration has not been reached. NaOH is the limiting reagent. The

Henderson-Hasselbalch equation can be used to solve for x. Because HF and F− are contained in the

same volume of solution, we can plug in moles into the Henderson-Hasselbalch equation to solve for x.

The Ka value for HF is 7.1 × 10−4

.

a[F ]

pH p log[HF]

= +K

2.85 3.15 log0.0050

= +

x

x

0.3010

0.0050

−=

x

x

0.0025 − 0.50x = x

x = 0.00167 mol

The volume of NaOH added can be calculated from the molarity of NaOH and moles of NaOH.

1 L

0.00167 mol NaOH0.20 mol NaOH

× = =0.0084 L 8.4 mL

(b) The pKa value of HF is 3.15. This is halfway to the equivalence point of the titration, where the

concentration of HF equals the concentration of F−. For the concentrations of HF and F

− to be equal,

12.5 mL of 0.20 M NaOH must be added to 25.0 mL of 0.20 M HF.

HF(aq) + NaOH(aq) → NaF(aq) + H2O(l)

Initial (mol): 0.0050 0.0025 0

Change (mol): −0.0025 −0.0025 +0.0025

Final (mol) 0.0025 0 0.0025

Page 56: Chapter 16 solution manual 11e

CHAPTER 16: ACID-BASE EQUILIBRIA AND SOLUBILITY EQUILIBRIA 597

(c) At pH = 11.89, the equivalence point of the titration has been passed. Excess OH− ions are present in

solution. First, let’s calculate the [OH−] at pH = 11.89.

pOH = 2.11

[OH−] = 10

−pOH = 0.0078 M

Because the initial concentrations of HF and NaOH are equal, it would take 25.0 mL of NaOH to reach

the equivalence point. Therefore, the total solution volume at the equivalence point is 50.0 mL. At the

equivalence point, there are no hydroxide ions present in solution. After the equivalence point, excess

NaOH is added. Let x equal the volume of NaOH added after the equivalence point. A NaOH solution

with a concentration of 0.20 M is being added, and a pH = 11.89 solution has a hydroxide concentration

of 0.0078 M. A dilution calculation can be set up to solve for x.

M1V1 = M2V2

(0.20 M)(x) = (0.0078 M)(50 mL + x)

0.192x = 0.39

x = 2.0 mL

This volume of 2.0 mL NaOH solution is the volume added after the equivalence point. The total

volume of NaOH added to reach a pH of 11.89 is:

volume NaOH = 25.0 mL + 2.0 mL = 27.0 mL

16.135

1apH p 6.38= =K 2apH p 10.32= =K

Main features: At low pH’s, H2CO3 predominates and 2 3 3H CO / HCO−are the only important species. At high

pH’s,23CO −

predominates and2

3 3HCO / CO− −are the only important species. Also, at fraction 0.5, we have

2 3 3[H CO ] [HCO ]−= so

1apH p= K and2

3 3[HCO ] [CO ]− −= so

2apH p .= K

0.0

0.5

1.0

0 1 2 3 4 5 6 7 8 9 10 11 12 13 14

Fra

cti

on

of

sp

ec

ies

pre

se

nt

pH

H2CO3 CO32−

HCO3−

Page 57: Chapter 16 solution manual 11e

CHAPTER 16: ACID-BASE EQUILIBRIA AND SOLUBILITY EQUILIBRIA 598

16.136 (a) Before dilution:

3a

3

[CH COO ]pH p log

[CH COOH]

= +K

[0.500]

4.74 log[0.500]

= + =pH 4.74

After a 10-fold dilution:

[0.0500]

4.74 log[0.0500]

= + =pH 4.74

There is no change in the pH of a buffer upon dilution.

(b) Before dilution:

CH3COOH(aq) + H2O(l) ⇌ H3O+(aq) + CH3COO

−(aq)

Initial (M): 0.500 0 0

Change (M): −x +x +x

Equilibrium (M): 0.500 − x x x

3 3a

3

[H O ][CH COO ]

[CH COOH]

+ −

=K

2 2

51.8 100.500 0.500

−× = ≈

x x

x

x = 3.0 × 10−3

M = [H3O+]

pH = −log(3.0 × 10−3

) = 2.52

After dilution:

2 2

51.8 100.0500 0.0500

−× = ≈

x x

x

x = 9.5 × 10−4

M = [H3O+]

pH = −log(9.5 × 10−4

) = 3.02

Page 58: Chapter 16 solution manual 11e

CHAPTER 16: ACID-BASE EQUILIBRIA AND SOLUBILITY EQUILIBRIA 599

16.137

H3N CH C

CH2

O

O

HN

NH

+ −H3N CH C

CH2

OH

O

HN

NH

+

+ +

H3N CH C

CH2

O

O

N

NH

+ −H2N CH C

CH2

O

O

N

NH

pKa = 1.82

pKa = 9.17

pKa = 6.00

A B

C D

(a)

1 2

3

(b) The species labeled C is the dipolar ion because it has an equal number of + and − charges.

(c) 2 3a ap p 6.00 9.17p

2 2

+ += = = 7.59

K KI

(d) Because the pH of blood is close to 7, the conjugate acid-base pair most suited to buffer blood is B (acid)

and C (base), because 2apK is closest to 7.

16.138 The reaction is:

NH3 + HCl → NH4Cl

First, we calculate moles of HCl and NH3.

HCl

1 atm372 mmHg (0.96 L)

760 mmHg0.0194 mol

L atm0.0821 (295 K)

mol K

×

= = =

PVn

RT

3

3NH

0.57 mol NH0.034 L 0.0194 mol

1 L soln= × =n

Page 59: Chapter 16 solution manual 11e

CHAPTER 16: ACID-BASE EQUILIBRIA AND SOLUBILITY EQUILIBRIA 600

The mole ratio between NH3 and HCl is 1:1, so we have complete neutralization.

NH3 + HCl → NH4Cl

Initial (mol): 0.0194 0.0194 0

Change (mol): −0.0194 −0.0194 +0.0194

Final (mol): 0 0 0.0194

4NH+is a weak acid. We set up the reaction representing the hydrolysis of 4NH .+

4NH ( )+aq + H2O(l) ⇌ H3O

+(aq) + NH3(aq)

Initial (M): 0.0194 mol/0.034 L 0 0

Change (M): −x +x +x

Equilibrium (M): 0.57 − x x x

3 3a

4

[H O ][NH ]

[NH ]

+

+=K

2 2

105.6 100.57 0.57

−× = ≈

x x

x

x = 1.79 × 10−5

M = [H3O+]

pH = −log(1.79 × 10−5

) = 4.75

16.139 (a) Assuming complete dissociation and no ion-pair formation, the concentration of ions in solution is

0.50 m × 2 = 1.00 m. The freezing point of the solution is:

∆Tf = Kfm = (1.86°C/m)(1.00 m) = 1.86°C

freezing point = 0 °C −−−− 1.86°C = −−−−1.86°°°°C

(b) HgI2 + 2I− →

24HgI −

Assuming that the all the free I− ions are removed from the solution, and the volume of the solution

remains constant, the concentration of 24HgI −

is one-half that of I−. The concentration of ions in

solution is now 0.75 m (0.50 m Na+, 0.25 m

24HgI −

).

∆Tf = Kfm = (1.86°C/m)(0.75 m) = 1.4°C

freezing point = 0 °C −−−− 1.4°C = −−−−1.4°°°°C

16.140 (a) The potential precipitate is MgCO3. We solve for Q = Ksp to determine the maximum mass of Na2CO3

that can be added without MgCO3 precipitating. The concentration of Mg2+

ions is 0.100 M.

Q = [Mg2+

]23[CO ]−

4.0 × 10−5

= (0.100 M)

23[CO ]−

23[CO ]−

= 4.0 × 10−4

M

4 23 2 3 2 3

22 33

4.0 10 mol CO 1 mol Na CO 105.99 g Na CO0.200 L

1 L soln 1 mol Na CO1 mol CO

− −

×= × × × =2 3mass Na CO 0.0085 g

Page 60: Chapter 16 solution manual 11e

CHAPTER 16: ACID-BASE EQUILIBRIA AND SOLUBILITY EQUILIBRIA 601

(b) The potential precipitate is AgCl. We solve for Q = Ksp to determine the maximum mass of AgNO3

that can be added without AgCl precipitating. The concentration of Cl− ions is 0.200 M because 1 mole

of MgCl2 yields 2 moles of Cl−.

Q = [Ag+][Cl

−]

1.6 × 10−10

= [Ag+](0.200 M)

[Ag+] = 8.0 × 10

−10 M

103 3

3

1 mol AgNO 169.9 g AgNO8.00 10 mol Ag0.200 L

1 L soln 1 mol AgNO1 mol Ag

− +

+

×= × × × =

83mass AgNO 2.7 10 g−−−−××××

(c) The potential precipitate is Mg(OH)2. We solve for Q = Ksp to determine the maximum mass of KOH

that can be added without Mg(OH)2 precipitating. The concentration of Mg2+

ions is 0.100 M.

Q = [Mg2+

][OH−]2

1.2 × 10−11

= (0.100 M)[OH−]2

[OH−] = 1.1 × 10

−5 M

51.1 10 mol OH 1 mol KOH 56.11 g KOH

0.200 L1 L soln 1 mol KOH1 mol OH

− −

×= × × × =

4mass KOH 1.2 10 g−−−−××××

16.141 The reaction is:

Ag2CO3(aq) + 2HCl(aq) → 2AgCl(s) + CO2(g) + H2O(l)

The moles of CO2(g) produced will equal the moles of23CO −

in Ag2CO3 due to the stoichiometry of the reaction.

2

4CO

1 atm114 mmHg (0.019 L)

760 mmHg1.16 10 mol

L atm0.0821 (298 K)

mol K

×

= = = ×

PVn

RT

Because the solution volume is 1.0 L,23[CO ]−

= 1.16 × 10−4

M. We set up a table representing the dissociation of

Ag2CO3 to solve for Ksp.

Ag2CO3 ⇌ 2Ag+ +

23CO −

Equilibrium (M): 2(1.16 × 10−4

) 1.16 × 10−4

Ksp = [Ag+]2 2

3[CO ]−

Ksp = (2.32 × 10−4

)2(1.16 × 10

−4)

Ksp = 6.2 × 10−−−−12

(at 5°C)

16.142 (1) The initial pH of acid (a) is lower.

(2) The pH at the half-way to the equivalence point is lower for acid (a). At the half-way point, pH = pKa.

The Ka value of the first acid is therefore larger.

(3) The pH at the equivalence point is lower for acid (a). At this point, the pH is determined by the

hydrolysis of the conjugate base of the weak acid. A lower pH indicates a weaker conjugate base and

hence a stronger acid.

Based on these three observations, acid (a) is the stronger acid.

Page 61: Chapter 16 solution manual 11e

CHAPTER 16: ACID-BASE EQUILIBRIA AND SOLUBILITY EQUILIBRIA 602

16.143 (1) The initial pH of the base (a) is higher.

(2) The pH at the half-way to the equivalence point is higher for base (a). At the half-way point, pH = pKa.

A higher pKa value means that the Ka value of the conjugate acid is smaller, and hence the Kb value is

larger (Ka × Kb = Kw).

(3) The pH at the equivalence point is higher for base (a). At this point, the pH is determined by the

hydrolysis of the conjugate acid of the weak base. A higher pH indicates a weaker conjugate acid and

hence a stronger base.

Based on these three observations, base (a) is the stronger base.

16.144 When the CuSO4 solution is mixed with the Ba(OH)2 solution, both Cu(OH)2 and BaSO4 precipitate from

solution. The solutions are mixed in the correct stoichiometric ratio. If we calculate Q (the reaction quotient)

for each of these salts, we would find that Q > Ksp, indicating that both salts precipitate.

CuSO4(aq) + Ba(OH)2(aq) → Cu(OH)2(s) + BaSO4(s)

Initial (mol): 0.0100 0.0100 0.00 0.00

Change (mol): −0.0100 −0.0100 +0.0100 +0.0100

Final (mol): 0 0 0.0100 0.0100

We can now consider the dissociation of each precipitate in water to determine the concentration of ions in

the combined solution.

Cu(OH)2(s) ⇌ Cu2+

(aq) + 2OH−(aq)

Initial (M): 0 0

Change (M): −s +s +2s

Equilibrium (M): s 2s

Recall that the concentration of a pure solid does not enter into an equilibrium constant expression.

Therefore, the amount of Cu(OH)2 is unimportant as long as precipitate is present.

Ksp = [Cu2+

][OH−]2

2.2 × 10−20

= (s)(2s)2

2.2 × 10−20

= 4s3

s = 1.8 × 10−7

M

[Cu2+

] = s = 1.8 × 10−7

M

[OH−] = 2s = 3.6 × 10

−7 M

BaSO4(s) ⇌ Ba2+

(aq) + 24SO −

(aq)

Initial (M): 0 0

Change (M): −s +s +s

Equilibrium (M): s s

Ksp = [Ba2+

]24[SO ]−

1.1 × 10−10

= s2

s = 1.0 × 10−5

M

[Ba2+

] = 24[SO ]−−−− = s = 1.0 × 10

−5 M

Page 62: Chapter 16 solution manual 11e

CHAPTER 16: ACID-BASE EQUILIBRIA AND SOLUBILITY EQUILIBRIA 603

16.145 From left to right. (1) First point. This is the half equivalence point for the first ionization. We have

1apH p 4.8= ≈K . (2) Second point. This is the equivalence point for the first ionization. pH ≈ 7. (3) Third

point. This is the half equivalence point for the second ionization. 2apH p 9.0= ≈K . (4) Fourth point. This is

the equivalence point for the second ionization. pH ≈ 10.6. (5) Fifth point. At this point, both ionizable

protons have been removed. The pH of the solution is mainly determined by the excess NaOH present.

16.146 From left to right. (1) This is the half equivalence point for the first base ionization.

1apH p 11.0= ≈K so

1 1b ap 14.00 p 14.00 11.0 3.0= − = − ≈K K . (2) Second point. This is the equivalence point for the first base

ionization. pH ≈ 8.4. (3) Third point. This is the half equivalence point for the second base ionization.

2apH p 6.0= ≈K so 2 2b ap 14.00 p 14.00 6.0 8.0= − = − ≈K K . (4) Fourth point. This is the equivalence

point for the second base ionization. pH ≈ 3.8. (5) Fifth point. At this point, both base ionizations have taken

place (that is, both –NH2 groups are converted to 3NH+− groups) and the pH of the solution is mainly

determined by the excess HCl present.

16.147 As the graph shows, aluminum hydroxide is totally insoluble between pH 4 and pH 10. However, being an

amphoteric hydroxide, it reacts with acids at low pHs and bases at high pHs which account for its solubility.

Below pH = 4, the solubility equilibrium equation is

Al(OH)3(s) + 3H3O+(aq) ⇌ Al

3+(aq) + 6H2O(l)

Above pH = 10, the solubility equilibrium equation is

Al(OH)3(s) + OH–(aq) ⇌ 4Al(OH) ( )−

aq

16.148 Basic approach:

• Consider the form of the equation relating molar solubility to solubility product.

To compare the solubilities of two ionic compounds, they must have similar formulas; that is, both must be

XY or XY2, etc. This condition is not met here because Ag+ and Fe

2+ bear different charges. We can

calculate the molar solubilities using the formulas in Table 16.3.

AgBr

–13 –7sp 7.7 10 8.8 10 mol/L= = × = ×s K

Fe(OH)2

1/31/3 –14sp –51.6 10

1.6 10 mol/L4 4

×= = = ×

Ks

Thus, Fe(OH)2 is more soluble than AgBr even though it has a smaller Ksp value. When making ballpark

estimates of solubility, we must remember that only when ionic compounds have similar formulas can we

compare their solubilities based on their Ksp values.

16.149 Basic approach:

• Look up the pH of the biological fluids (see Table 15.1).

• Solve for ratio of neutral to deprotonated aspirin using the Henderson-Hasselbalch equation.

The pH for the body fluids listed in this problem can be found online, but they are also given in Table 15.1.

Page 63: Chapter 16 solution manual 11e

CHAPTER 16: ACID-BASE EQUILIBRIA AND SOLUBILITY EQUILIBRIA 604

The Henderson-Hasselbalch equation

a[conjugate base]

pH p + log[acid]

= K

can be rearranged to give

ap pH[acid] [neutral aspirin]10

[conjugate base] [deprotonated aspirin]

−= =

K

permitting ready calculation of the ratio of neutral to deprotonated aspirin.

(a) saliva, pH = 6.4-6.9

Using pH = 6.7 gives 3.5 6.7 4[neutral aspirin]

10 6.3 10[deprotonated aspirin]

− −= = ×

(b) gastric juices in the stomach, pH = 1.0-2.0

Using pH = 1.5 gives 3.5 1.5 2[neutral aspirin]

10 1.0 10[deprotonated aspirin]

−= = ×

(c) blood stream, pH = 7.35-7.45

Using pH = 7.4 gives 3.5 7.4 4[neutral aspirin]

10 1.3 10[deprotonated aspirin]

− −= = ×

Note how the ratio of neutral to deprotonated aspirin varies dramatically with the pH of the body fluid in

which it is dissolved. This ratio is important in pharmacology when considering the transport of drugs

between the digestive system, the blood stream, and other biological tissues.

Answers to Review of Concepts

Section 16.3 (p. 730) (a) and (c) can act as buffers. (c) has a greater buffer capacity.

Section 16.4 (p. 741) (a), (c), and (d).

Section 16.5 (p. 744) The end point of an acid-base titration will accurately represent the equivalence point when

the pH region over which the indicator changes color matches the steep portion of the

titration curve.

Section 16.6 (p. 746) (b) Supersaturated. (c) Unsaturated. (d) Saturated.

Section 16.7 (p. 753) AgBr will precipitate first and Ag2SO4 will precipitate last.

Section 16.8 (p. 755) (a) NaClO3(aq). (b) Ba(NO3)2(aq).

Section 16.10 (p. 761) (c) KCN